Thursday, October 27, 2016

You can't tell a relativistic rest frame from measurements

So, after a year-and-a-half of no posts, and something like four years since my last post on relativity crankery, here we go again. Below the fold, I'm going to discuss whether seeing a smaller number on one clock versus another is an indication of which clock really moved in an LR interpretation. To do so, I'll use three scenarios, each with three clocks. The scenarios will be set up so that the clock which is "really at rest" changes between each scenario, but otherwise they will behave identically.



There are three clocks at rest on a planetoid, we will label them A1, B2, and C3. By whatever method you think applies, we'll say these three clocks ar all in the LR rest frame. They are close enough to synchronize without meaningful light-speed delay, and they do so. After synchronizing, they ignore each other, with A1 interacting with clocks B1 and C1, B2 interacting with clocks A2 and C2, and C3 interacting with clocks A3 and B3.


Scenario 1

From the viewpoint of A1, B1 passes by A1 (spacetime point S1) going at .5c; both set themselves to 0. C1 is traveling at .5c directly toward A1 along a path very close to B1 in the opposite direction. After traveling 10 ls (light-seconds) at this speed, B1 and C1 pass each other (spacetime point R1) and synchronize. B1 traveled for 20 seconds, and ticked off 17.32 seconds, between S1 and R1, so click C1, which has clicked through 17.32 seconds in this time, will also read 17.32 because of this synchronization. C1 ticks off 17.32 more seconds on the trip between R1 and the point where it compares itself with A1 (spacetime point Q1), while A1 ticks off another 20. So, C1 reads 34.64 seconds, A1 reads 40. Since B1 and C1 ticks at the same rate, B1 also reads 34.64 seconds, but since B1 is many light-seconds away from Q1, it can not be directly compared to A1 and C1.

From the viewpoint of B1, A1 passes by B1 (S1) going at .5c; both set themselves to 0. C1 is traveling at .8c directly toward B1 along a path very close to A1 in the same direction. A1 travels 8.66 ls in 17.32 seconds before C1 passes B1 (R1). Even though it has only clicked through 10.39 seconds, C1 synchronizes to the 17.32 seconds on B. In another 28.86 seconds (the solution to .8t = .5t + 8.66), C1 passes A1, and they compare times (Q1). In traveling 28.86 seconds at .8c, C1 has ticked off another 17.32 seconds, and reads 34.64. Meanwhile, A1 has traveled at .5c for 46.18 seconds (for a total distance of 23.09 ls), and reads 40. So, C1 reads 34.64 seconds (despite having clicked through only 27.71), A1 reads 40. B1 reads 46.18 seconds, but since B1 is many light-seconds away from Q1, it can not be directly compared to A1 and C1.

From the viewpoint of C1, B1 passes by A1 (S1) at a distance of 23.09ls away, B1 is coming toward C1 at a speed of .8c, and both A1 and B1 set themselves to 0 at S1. A1 is traveling at .5c directly toward C1 along a path very close to B1 in the same direction. B1 travels 23.09 ls in 28.86 seconds before it passes C1 (R1), and C1 then synchronizes to the B1's reading of 17.32 seconds. In another 17.32 seconds, A1 passes C1, and they compare times (Q1). In traveling 46.18 seconds at .5c, A1 has ticked off 40 seconds, and C1 reads 34.64 because of the synchronization at R1. Meanwhile, B1 has traveled at .8c for 46.18 seconds, and reads 27.71 seconds. So, C1 reads 34.64 seconds even though it ticked off 46.18, A1 reads 40. B1 reads 27.71 seconds, but since B1 is many light-seconds away from Q1, it can not be directly compared to A1 and C1.


Scenario 2

From the viewpoint of A2, B2 passes by A2 (spacetime point S2) going at .5c; both set themselves to 0. C2 is traveling at .5c directly toward A2 along a path very close to B2 in the opposite direction. After traveling 10 ls (light-seconds) at this speed, B2 and C2 pass each other (spacetime point R2) and synchronize. B2 traveled for 20 seconds, and ticked off 17.32 seconds, between S2 and R2, so click C2, which has clicked through 17.32 seconds in this time, will also read 17.32 because of this synchronization. C2 ticks off 17.32 more seconds on the trip between R2 and the point where it compares itself with A2 (spacetime point Q2), while A2 ticks off another 20. So, C2 reads 34.64 seconds, A2 reads 40. Since B2 and C2 ticks at the same rate, B2 also reads 34.64 seconds, but since B2 is many light-seconds away from Q2, it can not be directly compared to A2 and C2.

From the viewpoint of B2, A2 passes by B2 (S2) going at .5c; both set themselves to 0. C2 is traveling at .8c directly toward B2 along a path very close to A2 in the same direction. A2 travels 8.66 ls in 17.32 seconds before C2 passes B2 (R2). Even though it has only clicked through 10.39 seconds, C2 synchronizes to the 17.32 seconds on B. In another 28.86 seconds (the solution to .8t = .5t + 8.66), C2 passes A2, and they compare times (Q2). In traveling 28.86 seconds at .8c, C2 has ticked off another 17.32 seconds, and reads 34.64. Meanwhile, A2 has traveled at .5c for 46.18 seconds (for a total distance of 23.09 ls), and reads 40. So, C2 reads 34.64 seconds (despite having clicked through only 27.71), A2 reads 40. B2 reads 46.18 seconds, but since B2 is many light-seconds away from Q2, it can not be directly compared to A2 and C2.

From the viewpoint of C2, B2 passes by A2 (S2) at a distance of 23.09ls away, B2 is coming toward C2 at a speed of .8c, and both A2 and B2 set themselves to 0 at S2. A2 is traveling at .5c directly toward C2 along a path very close to B2 in the same direction. B2 travels 23.09 ls in 28.86 seconds before it passes C2 (R2), and C2 then synchronizes to the B2's reading of 17.32 seconds. In another 17.32 seconds, A2 passes C2, and they compare times (Q2). In traveling 46.18 seconds at .5c, A2 has ticked off 40 seconds, and C2 reads 34.64 because of the synchronization at R2. Meanwhile, B2 has traveled at .8c for 46.18 seconds, and reads 27.71 seconds. So, C2 reads 34.64 seconds even though it ticked off 46.18, A2 reads 40. B2 reads 27.71 seconds, but since B2 is many light-seconds away from Q2, it can not be directly compared to A2 and C2.


Scenario 3

From the viewpoint of A3, B3 passes by A3 (spacetime point S3) going at .5c; both set themselves to 0. C3 is traveling at .5c directly toward A3 along a path very close to B3 in the opposite direction. After traveling 10 ls (light-seconds) at this speed, B3 and C3 pass each other (spacetime point R3) and synchronize. B3 traveled for 20 seconds, and ticked off 17.32 seconds, between S3 and R3, so click C3, which has clicked through 17.32 seconds in this time, will also read 17.32 because of this synchronization. C3 ticks off 17.32 more seconds on the trip between R3 and the point where it compares itself with A3 (spacetime point Q3), while A3 ticks off another 20. So, C3 reads 34.64 seconds, A3 reads 40. Since B3 and C3 ticks at the same rate, B3 also reads 34.64 seconds, but since B3 is many light-seconds away from Q3, it can not be directly compared to A3 and C3.

From the viewpoint of B3, A3 passes by B3 (S3) going at .5c; both set themselves to 0. C3 is traveling at .8c directly toward B3 along a path very close to A3 in the same direction. A3 travels 8.66 ls in 17.32 seconds before C3 passes B3 (R3). Even though it has only clicked through 10.39 seconds, C3 synchronizes to the 17.32 seconds on B. In another 28.86 seconds (the solution to .8t = .5t + 8.66), C3 passes A3, and they compare times (Q3). In traveling 28.86 seconds at .8c, C3 has ticked off another 17.32 seconds, and reads 34.64. Meanwhile, A3 has traveled at .5c for 46.18 seconds (for a total distance of 23.09 ls), and reads 40. So, C3 reads 34.64 seconds (despite having clicked through only 27.71), A3 reads 40. B3 reads 46.18 seconds, but since B3 is many light-seconds away from Q3, it can not be directly compared to A3 and C3.

From the viewpoint of C3, B3 passes by A3 (S3) at a distance of 23.09ls away, B3 is coming toward C3 at a speed of .8c, and both A3 and B3 set themselves to 0 at S3. A3 is traveling at .5c directly toward C3 along a path very close to B3 in the same direction. B3 travels 23.09 ls in 28.86 seconds before it passes C3 (R3), and C3 then synchronizes to the B3's reading of 17.32 seconds. In another 17.32 seconds, A3 passes C3, and they compare times (Q3). In traveling 46.18 seconds at .5c, A3 has ticked off 40 seconds, and C3 reads 34.64 because of the synchronization at R3. Meanwhile, B3 has traveled at .8c for 46.18 seconds, and reads 27.71 seconds. So, C3 reads 34.64 seconds even though it ticked off 46.18, A3 reads 40. B3 reads 27.71 seconds, but since B3 is many light-seconds away from Q3, it can not be directly compared to A3 and C3.

Conclusion

In scenario 1, where A1 was at rest, A1 read 40 seconds while C1 read 34.64 at Q1. In scenario 2, where B2 was at rest, A2 read 40 seconds while C2 read 34.64 at Q2. In scenario 3, where C3 was at rest, A3 read 40 seconds while C3 read 34.64 at Q3. Among the three clocks, it doesn't matter which clock you think is at rest. They always read the same. This is how SR (and LR for that matter)works. To do comparisons, you always have to pick a single rest frame for your calculations, but SR allows you to choose any single inertial frame, by any standard or by a purely arbitrary choice.

LR insists that one particular frame is the correct frame, and different versions of LR choose different frames. Despite choosing frames differently, every version of LR works, because every version of LR is actually SR (a working theory) with a predetermined method for choosing a frame.


123 comments:

One Brow said...

If both A and B see A's clock as being 40 and B's as being 34.64,

Since they don't, I'll just ignore the part where you talk as if they do, unless there is somethig worth addressing.

=====

"1. "In all three frames, clock C reads 34.64, clock A reads ."

Which one is it, this time?


It's the one where B and C don't necessarily read the same, becuase of the relativity of simultaneity.

Again, assuming this is "true," can't you see that it contradicts the basic claim of SR that "each observer will "see" the other's clock as going slower?"

"I don't see what is not true."

======

If you ever actually understand this, then maybe we can make some progress on this topic.


Back atcha.

"A path is the amount of distance you use to go from point A to point B. The distance doesn't change, but the length of a path can change."

Do you mean distance, or that distortion of distance relativists call"spacetime"

This is incoherent, as stated, if "lenghth of a path" is supposed to mean "distance," instead of sometime else, like time maybe.


I can just see you returning a rental car. You had 50 miles a day for free, but you drove to KC and back over the weekend. The lady tries to charge you for extra mileage, but you say "I only drove 30 feet. See, that car was in that space when I left, and I parked it in this space, and they are only 30 feet apart. The car only traveled 30 feet."

Given how you want to define distance, distance is a constant. However, path length can change.

His point is quite clear, and has nothing to do with "the earth," per se.

Frankly, I don't care. Telling me how important the selection of a particular reference frame is for LR is like telling me how the important the color of your car, because you want the best gas mileage. First, you have to show me where there is a difference in the metaphorical mileage.

By the way, do you get that an etalon-based LR is different from a local-gravitational-field LR? You presenting two different variations of LR as if both are true. That's very SR-like of you.

He doesn't actually say that SR doesn't "allow for it," but he does say, like Hogg, that the changes must be real, and not just illusory or just a "matter of perspective."

Changes in perspective (that is, physical positioning) are not real? A solar eclipse is a different perspective on the relative location of the moon and sun from the surface of the earth. Solar eclipses are not real?

This time, can you come up with a better answer than 'a solar eclipse doesn't change the nature of the sun, moon, or earth'? That's my point. A change in perspective is a real change, but not a change in any of the constituent parts.

One Brow said...

"Mr. Percival seems to have missed that SR predicts that a GPS satellite clock in orbit would be predicted to "go slower" than a clock with the tangential velocity of a GPS satellite clock in orbit."

I have no idea what you're trying to say here, and obviously you don't understand what he's trying to say, as obvious and simple as it is.


I understand what he's trying to say. He's just wrong. He's trying to say that since the orbital speed of the satellite is greater, so supposedly SR would say the earth clock would be slower. However, he remembers enough physics to know that what matters in SR is velocity, so he wants to discuss an orbital velocity. Except, velocity is both a speed and a direction, and a satellite does not keep moving in the same direction, so to get a velocity, you need to use a straight-line construction, like a tangential velocity. If you are in a tangential orbital velocity (TOV), and you measure a clock on the surface of the earth, that clock will be slowed.

However, satellites are not moving in TOVs. They are actually in orbit, which is different from a TOV. In fact, for much of the orbit, the difference in velocity between the satellite and any specific TOV will be larger than the difference between an object on the surface of the earth and that TOV. Thus, overall, the satellite is slowed even more compared to the TOV than an object on the surface of the earth. SR allows you to use either the ECI or any individual TOV; regardless of choice, the satellite is slowed more. One flavor of LR says you have to choose the ECI, and then the satellite is slowed more. LR offers no advantage for this restriction.

1. You always want it both ways, don't you? "SR predicts that a GPS satellite clock in orbit would be predicted to "go slower" than a clock with the tangential velocity of a GPS satellite clock in orbit."

Just a minute ago you were insisting that SR makes exactly "no predictions" because it doesn't apply in non-inertial frames, eh?


A TOV is an inertial frame.

2. In SR, the satellite's "frame," if viewed as inertial, would have to say that ALL other clocks are "going slower" than it.

A satellite's frame is not inertial. It's a geodesic, but that's a GR identification, using notions that are not within the SR concepts contained in GR.

Physics uses, and has ALWAYS used, LR, not SR, when dealing with accelerating frames. LR has ALWAYS made accurate predictions in these cases while SR NEVER has.

Since they make the exact same predictions (as in my example), this will be hard for you to sensibly defend.

Here is it again, with you right back to claiming (contrary to your "standard response") that SR "makes predictions" about accelerating frames.

Which one will it be, next time?


LR requires the choice of an specific inertial frame (your sources have recommended either the ECI or the etalon frame, but either way, an inertial frame). SR requires the choice of any convenient inertial frame.

"If you say the HK experiment or the GPS, I will respond that every mainstream source sees these results as a confirmation of SR/GR. Find something that has a direct comparison, where there could be a real difference in the numbers read."

How could the GPS "confirm" SR, when, according to you, it does not even apply?


I said SR/GR, not just SR. SR is a subset of GR.

One Brow said...

My turn to repeat:
"In the example in this post (the two directly approaching each other), has Jill seeing Jack's clock tick off two seconds for every one she sees tick off on her own clock. Jack sees two seconds tick off on Jill's clock for every second on his. You can run an experiment on this in a lab for sensitive enough clocks."

Do you understand this is what happens?

====

What? We just went through this at great length. I denied that this is what happened, and said that's Jill's conclusions about Jack's other (synchronized it his frame) clock were DEDUCED, not "seen on her/his clock." You agreed, I thought. She DENIED what she did in fact see (which was only at the end of the journey, not "during" it.


I agreed that Fowler's description was indeed about what Jill deduced. That doesn't prevent Jill from looking through a window at Jack's clock; in fact, haven't you been consistently advocating that Jill needs to look outside the window? Similarly, it does not Jack from looking at Jill's clock. Since you are fond of LR, I will descibe all this from the framework of Jack.

Jack's clock and clock 1 are synchronized (in some absolutist fashion that really doesn't belong in Fowler's description to begin with), so that they both say 0 when Jill passes clock 1. However, Jack's clock is 6 ls away at this point, so Jill sees -6 on Jack's clock. As Jill moves toward Jack, her clock goes from 0 to 8 while her view of Jack's moves from -6 to 10. She gains 8, she sees Jack's gain 16. Jill sees Jack's clock gains 2 to her clock gaining 1.

Meanwhile, Jack sees Jill pass clock 1, both set to 0. However, he sees this 6 seconds after it happens. At this point, Jill is 6 seconds into her 10-second journey. By Jack's clock, it's just another 4 seconds until Jill passes Jack. In that time, his clock gains another four seconds, while Jill's clock moves from 0 to 8. He sees Jill's clock gain 2 seconds for every second his clock gains.

So, even in LR, each sees the other clock moving twice as fast. After you adjust for the light-speed delay, each measures the other clock to moving at .8 of their own clock's speed. Even under LR, there is a symmetry in measurement. There are no measurements Jack or Jill can make to say Jill is the one that is moving.

A. Without going into the details about how it would change our entire cosmology, interpretations of QM, etc., I just gave you an example where LR makes correct predictions where SR either a. makes none, or else, if applied, makes the wrong predictions. LR makes correction predictions in ACCELERATING frames, get that?

A proclamation is not an example. I'll take either a real-world experiment or a thought experiment. Show me the results of an actual experiment, or run through the numbers of your thought experiment, to show me the difference.

Again, if you choose real-world experiments like HK, I'll point out that scientists see them as confirmations of SR/GR, not refutations.

"I understand the passage, and dispute your interpretation."

Really? Just what, in what way, and why do you dispute it?


In that, since you get the same answer from SR as from LR, SR does indeed apply.

GR is not a theory of relative motion. It is irrelevant to shis topic, but you keep mentiioning it as a red herring. We're discussing LR vs SR.

If you are discussing SR, you are perforce discussing the theory that encompasses SR, which is GR. If somehow SR is disproven, so is GR.

One Brow said...

http://aether.lbl.gov/www/classes/p139/speed/space-time.html

More one-way speed drivel?

Did we not cover this before? If you move from point M to point N at speed v + x, and from N to M at speed v – x, the total time is different than moving at v.

For example, let's say the distance between M and N is 150 miles, v = 50 mph, x = 25 mph. It takes 3 hours each way moving at v, a total of 6 hours. Moving at 75 takes 2 hours, and at 25 takes 6 hours, for a total of 8 hours. If the speed of light was subject to Galilean relativity, you could detect this even when measuring a two-way speed.
Copyright 2007 by David Morin, morin@physics.harvard.edu

"One might view the statement, “A sees B’s clock running slow, and also B sees A’s clock running slow,” as somewhat unsettling. But in fact, it would be a complete disaster for the theory if A and B viewed each other in different ways. A critical fact in the theory of relativity is that A sees B in exactly the same way that B sees A."

http://www.people.fas.harvard.edu/~djmorin/chap11.pdf

A "complete disaster for SR," to assert YOUR view of it, eh?


My view is completely compatible with this. I enhanced it's description a little bit to empahsize that in A's frame, A measures clock A as gaining the most time; in B's frame, B measures clock B as gaining the most time, and in C's frame, C measures clock C as gaining the most time, yet in all three frames, at the comparison point C reads 34.64 and A reads 40, all of this regardless of whether A, B, or C is the one really at rest.

If all inertial observers reach the same conclusions in all inertial frames, there would be no SR.

The Lorentz equations feature a certain type of spacetime invariance. THis is a part of SR.

You don't understand the most fundamental premises of SR or their implications it seems. Clearly what the GPS demonstrates is this "complete disaster." The GPS satellites do not "see" the and ECI clocks as going slower than its own. They see them as going faster. The time dilation is NOT "reciprocal."

There are no ECI clocks, you would have to be at the center of the earth to be an ECI clock.

More importantly, the GPS satellites do not move in an inertial manner. There is nothing in SR preventing non-inertially moving observers from measuring other clocks as being slower than their clocks.

What is a "disaster" for SR is LR, which adopts the very premise that SR FORBIDS, and thereby makes the GPS possible.

SR does not forbid the premise of whichever flavor of LR you are promoting. It just doesn't feel the need to be tied down to it.

You say: "In all three frames, clock C reads 34.64, clock A reads 40"

That's simply saying that you KNOW one of them is wrong. That you KNOW that simultaneity is not "relative," that you KNOW that speed is absolute. And you think, that by saying this, you are "proving" SR. You are disproving it, that's all.


I am neither proving nor disproving SR, I'm explaining it.

Clocks tick off what they tick off. How is that wrong?

One Brow said...

More from Morin: Here he merely restates what I have already quoted the Baez physicist as saying:

"Our assumption that A is at rest on the train was critical in the above derivation. If A is moving with respect to the train, then eq. (11.9) doesn’t hold, because we cannot say that both A and B must agree. In everything we’ve done so far, we’ve assumed that A and B are in inertial frames, because these are the frames that the postulates of special relativity deal with....If, however, you are accelerating, then all bets are off, and it’s not valid for you to use the time dilation result when looking at a clock."

====

And Morin makes it clear that the travelling twin is the one who will be younger BECAUSE he is the one who is moving:


"If both A and B are blindfolded, they can still tell who is doing the traveling, because B will feel the acceleration at the turnaround."

Once again, the twin paradox just applies LR instead of SR to get it's resolution. In effect, it says the earth "really is" the preferred frame it this context because only that frame gives the correct answer. As between the two only one is "travelling," That can be determined and that twin will be younger, because the MOVING (not the non-moving) clock is the one (not both) that slows down.


Yet, when we convert to the three-clock scenario, you can assume any of them are not moving, and the numbers come out the same. You can rewrite the traveling twin so that one twin takes off, then the second takes off later, at a faster rate. The numbers come out the same. You can write it so that that a faster traveling twin lands first, then joined by a slower-traveling twin, the numbers come out the same.

To the extent the travelling "believed" or "assumed" that the earth clock was slowing down (which, incidentally he can do only be ignoring all observed empirical facts and instead slavishly adopteing a belief that SR it "true"), then, to that extent, he was just plain WRONG. That's because SR is wrong in it's most basic assumptions.

Within any inertial frame, including those which have the earth-bound twin moving the entire time and the traveling twin stationary for half the trip, the numbers come out the same as if you assume the earth-bound twin is stationary.

As Morin (and every other physicist, even if YOU don't) acknowledges, for SR to "work out" as a viable theory (and not face "complete disaster") it must suppose that each clock runs slower than the other. Of course this is logically impossible, so it's certainly no surprise that such a premise never comports with empirical observation.

Actually, it matches every empirical observation made from within inertial frames.

LR posits that, as a matter of objective reality, only one clock will "really" slow down.

In contrast, SR, the solipsistic theory, says that each clock runs slower than the other. I don't see how anyone could believe that is "literally" true, even if it does allow you to get correction prediction if and you when you posit either one clock or the other, as you desire, to be the one that is "really" moving.


Just as an object's measured speed is relative to your speed, an objects path through spacetime changes relative to your path through spacetime.

The resolution to the twin paradox does NOT say (because it can't say) that "both are right." Only the stay-at-home twin is right, they say. The travelling twin is wrong, because the earth clock did NOT, as he assumed, "actually" slow down.

Using either the outbound frame or the inbound frame, the traveling twin measures the earthbound clock to be slower on one part of the trip, and faster on the other part.

One Brow said...

2 x 2 = 4. Now I can get the "same answer" (4) mathematically, can by saying that -2 x -2 = 4. But getting the same answer does not mean both are, or could possibly be, true as a matter of fact. Me having $2 in one pocket and $2 in another is NOT the same thing (even if I get the same absolute answer when multiplying) as me not having a single cent, but owing you $2 and your brother $2. Both situations can't be true.

I agree multiplication is a bad model for that situation.

""In all three frames, clock C reads 34.64, clock A reads 40"


B, to A: In MY frame, your clock reads 40

A, to B: Really? Cool, that's what it reads in my frame too!

A & B: We have absolute simultaneity now. We agree. That's just what SR tells us....Uh, wait...that's not SR, that's LR.


Again, you convert C into B. Is that deliberate, or just careless?

In the three different frames, B has three different numbers.

He does say, later, however that it both SR and LR the "privileged" frame is the one where the etalons for time and distance are "at rest." It is my understanding that a frame that is "at rest" in an inertial frame. What do you think?

"He doesn't understand what an inertial frame is..." <---Heh.


Fair enough. He understand that the etalons need to be at rest. Does he offer a means (beside being in an inertial environment) to determine whether they are at rest? If not, this is standard SR.

I'm not going to bother to read the entire article, because I see no likely benefit. I have read several articles you have offered in the past; they always failed to live up to your hype. You are trying to make a point from the article; it's on you to at least summarize his point cogently. If it seems interesting after that, then I will read the article.

""In all three frames, clock C reads 34.64, clock A reads 40"

A to C: In MY frame, your clock reads 34.64 and mine reads 40. How do you read it?

C to A: Exactly the same way.

A and C: That's means we have been in the same inertial frame all this time, eh? I have no clue why we ever thought we were moving toward each other. Parmenides was right! All motion is an illusion.


??? If they were in the same inertial frame, both would rad 34.64 or both would read 40.

C to A: No it means that I was the one moving, that's all. My time slowed down, see? A: Well, that's better. Good to know we have an easy way to know who's moving faster and who isn't.

C's time did not slow down. From C's point of view, there was a total of 46.18 seconds between the start point and the comparison point. It is the synchronizing with B that causes C to show less time.

That's just what SR tells us! Uh, wait...that's not what SR tells us, that's LR. How about B? Does he get the same answers? C to A: He has to, because ALL inertial frames predict exactly the same thing, see!

B gets the same answers for A and C. A, B and C each get a different answer for B. However, B is too far away to compare with any of them directly.

One Brow said...

A: Yeah, I get it, he was moving slower than me too. He was going at exactly the same speed you were all this time. I don't know why anyone would ever think the two of you were moving at different speeds, relative to each other.

A: Actually, I'm getting kinda confused here. C: Me, too.


Anything I can do to clear up your confusion?

If someone tells me that 3 + 13 + 16 + 9 + 37 = 1,869, I don't even to add those numbers up to KNOW he is wrong, know what I'm sayin?

OK.

Nor would I have any idea how he came up with that ridiculous conclusion. Only HE could possibly know that.

I laid out a lot of my math. If there is something I did not explain, just ask.

If you're going 80 mph, how long does it take you to go 80 miles?

A: 8 eight hours.


I don’t find those videos funny. To each his own.

The essential difference between SR and LR:

LR says light is "Measured to be" constant in all inertial frames (but not accelerating frames)

SR says light *IS,* as a matter of ontology, constant in all inertial frames (but not accelerating frames)

So, the question becomes this: What is light "measured to be" in SR? Answer: It is it ALSO measured to be constant. This would seem to make perfect sense, since, according to SR, light *IS,* is fact constant. But there's one problem. SR ALSO posits that rod shrink and clocks slow down with speed. If that were true, AND IF the speed of light was in fact, as an ontological matter, "really" constant, then our "measurement" of it in an accelerated frame would have to be WRONG. We could not possibly measure it's "true" speed with distorted instruments.


The distortions are by the same factor (let’s call it g). Create to posts Y and Z in the same inertial frame with distance x. It takes light time t to move between them. The light moves at x/t. In another frame, the length of the posts is x', and the amount of time is t'. Since x' = xg, and t' = tg, you get x'/t' = (xg)/(tg) = x/t. The speed of light comes out the same.

Do you see a way around this problem, Eric? Let me guess--it's all illusory, right? "Time" slows down, not clocks. Can you explain how that works in terms of the objective world?

Time doesn't slow down. Time is not a physical object that can move more quickly or more slowly. Time is one dimension of the four-dimensional spacetime that you take a path through.

"Time" slows down, not clocks. Can you explain how that works in terms of the objective world?"

Also: Distance shrinks, not rods.

How does that work, exactly?


Distance doesn't expand/shrink. Space is not a physical object that can be larger or smaller. Space is 3 dimensions of the four-dimensional spacetime that you take a path through.

One Brow said...

The Lorentz Transformation, which assumes that rods shrink and clocks slow down with speed, i extremely well-tested and deemed to be true. It is so obviously "true" that SR uses it too. But, according to SR, as YOU interpret it, anyway, the LT is false.

Not at all. I interpret what the LT are saying differently that you, but I certainly accept their validity.

Tell me, then, since the LT is false, what is the formula for the "true" situation?

The LT describe the true situation.

What is the formula for the supposition that rods do NOT shrink at all, they remain completely unaltered, but instead distance shrinks and that clocks do NOT slow down, they remain completely unlatered, but instead time does.

I have no idea what such formulae would look like. The concept of time or distance changing seems just as untrue to me as the notion that rods and clocks undergo some internal, physical change that makes them shring and slow.

Ya got one?

Nope. I'm good with the LT.

"Do you see a way around this problem, Eric? Let me guess--it's all illusory, right? "Time" slows down, not clocks. Can you explain how that works in terms of the objective world?"

Before you are in with some nebulous metaphysical hocus pocus, let me explain what I mean by "in terms of the objective world."

The HK experiment, GPS, etc., have proven that clocks DO in fact slow down at increased speeds, as an objective, observable, physical matter.

Since that just happens to be "true," and not an illusion of some sort, address the contradiction first, OK?


It's interesting how we can be so critical and careful in our analysis of notions we question, but then just swallow whole stuff we are inclined to agree with. Here, you are seeing a clock that has more or less time than some other clock, and you immediately jump to the conclusion that it must be something affecting the internals to the clock or its functioning.

There is no contradiction. The clocks read having passed through a different amount of time, and there was no change to the clocks themselves nor anything that interfered with their mechanisms.

How can the "true" speed of light "really" be the same in all frames, yet STILL be measured to be the same in all frames with distorted instruments?

Discussed above.

One Brow said...

It's just one inherent contradiction with SR. It is (unanswerable) questions like these that lead many to claim that clock retardation and length contractions MUST be illusory.

In the SR that you describe, there are contradictions. In the SR used by physicists, they do not exist.

The changes are not illusory, nay more than shadows are an illusion.

There is a perfectly logical and consistent "resolution," but it ain't SR. It's LR, which makes perfect sense and has no inherent logical contradictions or "paradoxes."

Being a form of restricted SR, of course LR makes sense.

A solipsistic theory of physics, such as one that says that if two people assume opposite things, then each is right, is sheer folly. Al must have been REALLY desperate, eh? He has thrown objectivity and "truth" out the window; the baby with the bathwater. His "solution" is worse than the problem.

Unless the opposite things are a matter of perspective.

Well, in a way he did, I guess. I think it is commonly known and acknowledged that motion is not "relative" in GR and the speed of light is NOT constant in GR. This is ultimately a rejection of SR. Those who say GR "reduces" to SR in certain simplified situations overlook the fact the fundamental premises of SR are rejected in GR.

The fundamental premises specific to SR:

1. Identical experiments in inertial environments have identical results.
2. In particular, the speed of light in a vacuum moving inertially is always measured to be c.

The fundamental premises of GR:

1. Identical experiments in inertial environments have identical results.
2. In particular, the speed of light in a vacuum moving inertially is always measured to be c.
3. You can not distinguish the effects of gravity and the effects of acceleration on objects.

I don't see what you think is rejected.

One could just as easily say that, in inertial frames, LR can be "reduced" to SR.

Since LR is a restricted form of SR, absolutely. You can't do LR without being in an inertial frame. In fact, you may have missed this, but every version of LR you presenting contains a discussion of the proper inertial frame for LR.

aintnuthin said...

I was unaware that you were responding to comments in another thread here until just now. I can see immediately from reading your new post that you are utterly confused about the difference between SR and LR. You conflate the two, and hence don't understand the principles behind either of them. I'll come back to this mess later.

One Brow said...

I was unaware that you were responding to comments in another thread here until just now.

No worries. You can expect that every time we pass the 200 level in a thread. Comments had a tendency to get lost in the past when the thread covered more than one page. You can use that as an incentive to post fewer, longer comments, or don't. Same either way to me. I added a response to the rest of your post in the new thread.

One Brow said...

This is the new thread. Sorry about that.

I can see immediately from reading your new post that you are utterly confused about the difference between SR and LR. You conflate the two, and hence don't understand the principles behind either of them. I'll come back to this mess later.

I look forward to your listing and explanation of what you consider to be the principles of LR. As for SR, I am not confused. Those are basically the principles you will find in any mainstream discussions of the basis of SR. GR is not quite as simple as I made it out to be, although what I said is basically part of GR.

In your discussion, here's one thing you might keep in mind: an inertial frame is not just a coordinate system. For example, in the traveling twin scenario, you can create a coordinate system with the viewpoint that the traveling twin is not moving. It's just a mathematical construction, with no physical meaning. By contrast, an inertial frame is something you can detect physically by the lack of acceleration; it's not just a coordinate system.

aintnuthin said...

"I'm going to discuss whether seeing a smaller number on one clock versus another is an indication of which clock really moved in an LR interpretation."

The Lorentz transformations explicitly hold that the moving clock runs slower. This applies to both LR and SR. If you don't think it "really" holds, then you should write a new physics book, eh?

"The scenarios will be set up so that the clock which is "really at rest" changes between each scenario, but otherwise they will behave identically."

LR doesn't have multiple "stationary" points. Only SR does that. You are confusing the two.

aintnuthin said...

"...every version of LR is actually SR (a working theory)...."


Heh, proving conclusively that you have no idea of the conceptual bases of LR vs SR, despite me having explained it to you like 100 times.

aintnuthin said...

Scenario 1

From the viewpoint of A1, B1 passes by A1...From the viewpoint of B1, A1 passes by B1...rom the viewpoint of C1, B1 passes by A1..."

You'll never get it. All this "from the viewpoint of" crap is NOT LR, it's SR and ONLY SR.

In LR every observer "sees" things the same way, not a different way for each observer.

You should try to get a clue of what LR is about before pretentiously lecturing about it.

I'm afraid you're hopeless, Eric.

aintnuthin said...

He doesn't actually say that SR doesn't "allow for it," but he does say, like Hogg, that the changes must be real, and not just illusory or just a "matter of perspective."

"Changes in perspective (that is, physical positioning) are not real? A solar eclipse is a different perspective on the relative location of the moon and sun from the surface of the earth. Solar eclipses are not real?"

====

Read Hogg. I've quoted him at length in this thread, but I don't think you understood a word of it. You can't seem to get any kind of handle on what is subjective and what is objective. The mark of a true solipsist, sho nuff.

The "logic" you display here is completely invalid. As usual, you equivocate and treat different things as somehow being identical.

aintnuthin said...

Me: I just gave you an example where LR makes correct predictions where SR either a. makes none, or else, if applied, makes the wrong predictions. LR makes correction predictions in ACCELERATING frames, get that?

You: A proclamation is not an example. I'll take either a real-world experiment or a thought experiment. Show me the results of an actual experiment, or run through the numbers of your thought experiment, to show me the difference.

Again, if you choose real-world experiments like HK, I'll point out that scientists see them as confirmations of SR/GR, not refutations.

======

Wrong. "Scientists" see the HK as confirming the "relativistic effects of time dilation," Which means it confirms LR just as much as, or more so, than SR.

LR is used in the GPS. It has to. It could not POSSIBLY use SR, where every clock is slower than every other clock, to design an accurate, working system. LR was used to explain the observed results of HK. The fact that you don't understand this just proves, for the thousandth time, that you don't understand either SR or LR, and, a fortiori, don't understand the difference between them.

With respect to the HK you can't take one of two positions, I guess:

1. SR "applies" because the differences resulting from the earth's motion are minimal, and so SR makes the WRONG predictions about how the 3 different clocks will read when reunited, or

2. SR does NOT apply. So it was not used. So, what was used? LR, plain and simple.

Same with the GPS. Facts are facts, whether you like them or not, I'm afraid.

aintnuthin said...

There is a point with respect to which everything in the solar system is moving. It is not the sun. It is the barycenter, which the sun itself revolves around. This is the "preferred frame" for LR for making calculations regarding planetary motions (or other objects in the solar system).

You can't use the earth. You can't use mars. You can't use Venus.

Well, you can, but your predictions concerning time dilation would be entirely wrong. SR is NOT used for such calculations. If it were, then the Sun's clock would be slower than the earth's AND the earth's clock would be slower than the Sun's. Somehow that illogical postulation doesn't seem to render any results of practical use.

Newton said there was no way to detect some point which was absolutely motionless. Lorentz said the same. You seem to think, again, despite having it explained to you repeatedly, that LR requires the detection of "absolute" motion with respect to one universally applicable point. It doesn't. I doubt you'll ever get that. Once you get a notion in your head, it is impossible for change it. I must say that you are probably the most bigoted person I have ever encountered.

aintnuthin said...

The difference between 8 and 4 is 4. The difference between 27 and 23 is 4. The difference between 1,069 and 1,065 is 4. it's all the SAME DIFFERENCE, eh!?

So what does that prove? It's obvious, aint it? The numbers are all the same. They all give you the right answer. Therefore there can be no difference whatsoever between me having $1.069 (less 4) in the bank and me having $8 (less 4) in the bank.

What fool wouldn't understand that simple proof?

SR adherents understand it perfectly.

aintnuthin said...

Two guys, A & B, each exactly 6 feet tall, walk apart from each other for a certain distance. Then each one of them "appears" to be shorter.

They each get out their yardstick and "measure" how tall the other guy is at that distance. A determines that B is now only 5 feet tall. Guess what!?

B ALSO measures A to be only 5 feet tall!!!

All this while A and B both perceive themselves to be 6 feet tall!!

They get the exact same difference!

What can we conclude from this? Well, they're both only 5 feet tall, I guess. Or they're both 6 feet tall, it you prefer. Or, neither is 5 feet all and neither is 6 feet tall, because such things can't possibly be known, maybe.

One thing for sure though: each one is taller than the other.

aintnuthin said...

"Two guys, A & B, each exactly 6 feet tall, walk apart from each other for a certain distance. Then each one of them "appears" to be shorter."

Yes, the changed appearance is "real." But, no, each has not actually ("really") shrunk. I'm sure you don't understand this distinction though. To you, they're both "real."

aintnuthin said...

Somehow, though, in the HK experiment, each of the 3 clocks didn't manage to record less time elapsed than the other two. Go figure, eh?

aintnuthin said...

"Here, you are seeing a clock that has more or less time than some other clock, and you immediately jump to the conclusion that it must be something affecting the internals to the clock or its functioning.

There is no contradiction. The clocks read having passed through a different amount of time, and there was no change to the clocks themselves nor anything that interfered with their mechanisms."

Heh, are you aware of how clocks function? "simply passed through a different amount of time," eh? What the fuck does that mean? Time is a "thing" now, which you pass through, like a forest or something?

Do you ACTUALLY think that clocks measure a thing called "time." A THING? Time is a concept, not a "thing," sorry.

"Time" reaches into clocks and changes the position of the hands on the clock? Because, it is, after all, TIME?

You just can't see how ridiculous this assertion is, obviously.

aintnuthin said...

"SR allows you to use either the ECI or any individual TOV; regardless of choice, the satellite is slowed more. One flavor of LR says you have to choose the ECI, and then the satellite is slowed more. LR offers no advantage for this restriction."

Here again we revisit one of your most fundamental misconceptions. Nobody could convincingly claim to "understand" SR and even think about making such a claim.

You claim, with no basis whatsoever, that all frames of reference will predict the exact same amount of time dilation for all 3 clocks in the HK. Ridiculously wrong. This is the very claim that SR denies. But you think it is what SR claims.

Any (and every) frame of reference which is moving with respect to another will, by SR postulates, have a different prediction about this matter. They will agree on the difference in speed, but they will, and MUST (in SR) disagree about which clock has slowed down.

You seem to think that because they agree on relative speed, then they must agree about everything. The slightest amount of reflection would tell you otherwise, if you had even a cursory familiarity with SR, anyway.

aintnuthin said...

"You can't do LR without being in an inertial frame. In fact, you may have missed this, but every version of LR you presenting contains a discussion of the proper inertial frame for LR."

Of course you can. You can use LR to calculate the exact amount of time dilation he will experience both while he is blasting off (accelerating) and once he reaches "crusing" speed.

Here's the main difference between LR and SR. In SR, the astronaut will claim he is not moving (when cruising). In LR he will acknowledge that he is moving, and will understand and acknowledge that HIS clock (not the earth's) has slowed down compared to his.

I've said all this a thousand times. You still can't understand the difference between SR and LR.

They've sent chimpanzees into space. Maybe such as "astronaut" would think he is not moving toward the moon. But any human qualified to be an astronaut would NEVER think that. Only some confused SR advocate would claim that he does think that. Astronauts understand basic physics.

aintnuthin said...

"However, Jack's clock is 6 ls away at this point, so Jill sees -6 on Jack's clock. As Jill moves toward Jack, her clock goes from 0 to 8 while her view of Jack's moves from -6 to 10. She gains 8, she sees Jack's gain 16. Jill sees Jack's clock gains 2 to her clock gaining 1."

As Hogg (and any other decent physicists) makes clear, and as I have repeatedly pointed out to you, light speed delay has ABSOLUTELY NOTHING to do with time dilation or clock rates. Nor does the doppler effect. They are both totally irrelevant, and Hogg says you must CORRECT for this. You think that, because they "see" it, that makes it relevant? Fraid not, but here again, you just demonstrate your total inability to distinguish between subjective and objective factors. You also seem to have great trouble distinguishing non sequiturs from pertinent, relevant considerations.



T

aintnuthin said...

"light speed delay has ABSOLUTELY NOTHING to do with time dilation or clock rates. Nor does the doppler effect."

Fowler just demonstrates his unconscionable sophistry by bringing this kinda crap into the discussion to confuse the issue and purport to "prove" his mistaken point.

You just demonstrate your susceptibility to accepting fallacious sophistry. No wonder you are so prone to use it in your own arguments.

One Brow said...

"I'm going to discuss whether seeing a smaller number on one clock versus another is an indication of which clock really moved in an LR interpretation."

The Lorentz transformations explicitly hold that the moving clock runs slower. This applies to both LR and SR. If you don't think it "really" holds, then you should write a new physics book, eh?


I looked for some sign that you actually read the examples, looked at the numbers, and perhaps checked the calculations yourself. I did not see any such sign.

"The scenarios will be set up so that the clock which is "really at rest" changes between each scenario, but otherwise they will behave identically."

LR doesn't have multiple "stationary" points. Only SR does that. You are confusing the two.


I said, There are three clocks at rest on a planetoid, we will label them A1, B2, and C3. By whatever method you think applies, we'll say these three clocks are all in the LR rest frame.

A1, B2, and C3 are in the lone LR stationary frame. Try again.

"...every version of LR is actually SR (a working theory)...."

Heh, proving conclusively that you have no idea of the conceptual bases of LR vs SR, despite me having explained it to you like 100 times.


I listed the conceptual basis of SR, you have offered no correction. I await your list of the conceptual basis for LR.

Scenario 1

From the viewpoint of A1, B1 passes by A1...From the viewpoint of B1, A1 passes by B1...rom the viewpoint of C1, B1 passes by A1..."

You'll never get it. All this "from the viewpoint of" crap is NOT LR, it's SR and ONLY SR.


Everyone has a viewpoint, whether you consider measurements from that viewpoint to be accurate or not. The LT allows B1 and C1 to adjust their measurements to the frame of A1, but they still first make the measurement from their own respective viewpoints. The LT allows A2 and C2 to adjust their measurements to the frame of B2, but they still first make the measurement from their own respective viewpoints. The LT allows A3 and B3 to adjust their measurements to the frame of C3, but they still first make the measurement from their own respective viewpoints.

In LR every observer "sees" things the same way, not a different way for each observer.

No, in LR, every observer measures from their own viewpoint, and then uses the LT to correct those measurements to the preferred inertial frame.

You should try to get a clue of what LR is about before pretentiously lecturing about it.

I await your description of the basic principles of SR and LR.

I'm afraid you're hopeless, Eric.

Well, it seems unlikely I will wind up taking the viewpoint you espouse.

He doesn't actually say that SR doesn't "allow for it," but he does say, like Hogg, that the changes must be real, and not just illusory or just a "matter of perspective."

"Changes in perspective (that is, physical positioning) are not real? A solar eclipse is a different perspective on the relative location of the moon and sun from the surface of the earth. Solar eclipses are not real?"

====

Read Hogg. I've quoted him at length in this thread, but I don't think you understood a word of it. You can't seem to get any kind of handle on what is subjective and what is objective. The mark of a true solipsist, sho nuff.


You dodged rather than answering. Is a solar eclipse a real event (one which does not change the nature of the sun, moon, nor earth)?

One Brow said...

The "logic" you display here is completely invalid. As usual, you equivocate and treat different things as somehow being identical.

Be more specific. Where is the error in my post? Which of the measurements in the 3 scenarios does not happen?

Me: I just gave you an example where LR makes correct predictions where SR either a. makes none, or else, if applied, makes the wrong predictions. LR makes correction predictions in ACCELERATING frames, get that?

You: A proclamation is not an example. I'll take either a real-world experiment or a thought experiment. Show me the results of an actual experiment, or run through the numbers of your thought experiment, to show me the difference.

Again, if you choose real-world experiments like HK, I'll point out that scientists see them as confirmations of SR/GR, not refutations.

======

Wrong. "Scientists" see the HK as confirming the "relativistic effects of time dilation," Which means it confirms LR just as much as, or more so, than SR.


Certainly just as much as. SR and LR predict the same time dilation effects for the HK experiement. YOu can't use HK to differentiate between them.

LR is used in the GPS. It has to. It could not POSSIBLY use SR, where every clock is slower than every other clock, to design an accurate, working system.

Again, SR works from any inertial frame, including whichever frame you choose for LR. If LR works, SR, using the same equations in the same manner, also works. SR also works from any other inertial frame.

LR was used to explain the observed results of HK. The fact that you don't understand this just proves, for the thousandth time, that you don't understand either SR or LR, and, a fortiori, don't understand the difference between them.

https://en.wikipedia.org/wiki/Hafele%E2%80%93Keating_experiment
When reunited, the three sets of clocks were found to disagree with one another, and their differences were consistent with the predictions of special and general relativity.

This is the mainstream position, accepted by the vast majority of physicists. It's a confirmation of SR.

With respect to the HK you can't take one of two positions, I guess:

1. SR "applies" because the differences resulting from the earth's motion are minimal, and so SR makes the WRONG predictions about how the 3 different clocks will read when reunited, or

2. SR does NOT apply. So it was not used. So, what was used? LR, plain and simple.


Using LR *is* using SR. If SR did not apply, neither would LR.

Same with the GPS. Facts are facts, whether you like them or not, I'm afraid.

Doesn't seem to stop you.

One Brow said...

There is a point with respect to which everything in the solar system is moving. It is not the sun. It is the barycenter, which the sun itself revolves around. This is the "preferred frame" for LR for making calculations regarding planetary motions (or other objects in the solar system).

You can't use the earth. You can't use mars. You can't use Venus.


Agreed. In the scale of the solar system, Earth, Mars, and Venus are all in non-inertial motion, while the barycenter is in inertial motion. The barycenter is one of an infinite number of choices fo inertial frame, all of which work equally well.

Well, you can, but your predictions concerning time dilation would be entirely wrong. SR is NOT used for such calculations.

Again, SR works from any inertial frame, including whichever frame you choose for LR. If LR works, SR, using the same equations in the same manner, also works. SR also works from any other inertial frame.

If it were, then the Sun's clock would be slower than the earth's AND the earth's clock would be slower than the Sun's. Somehow that illogical postulation doesn't seem to render any results of practical use.

There is no SR calculation where the sun clock runs slower than the earth. If you really blelive otherwise, prove it. Run the actual numbers. Show me.

Newton said there was no way to detect some point which was absolutely motionless. Lorentz said the same. You seem to think, again, despite having it explained to you repeatedly, that LR requires the detection of "absolute" motion with respect to one universally applicable point. It doesn't.

Agreed. LR does not require the detection of an absolute motion. It only requires the selection of a single, preferred rest frame.

I doubt you'll ever get that.

You doubt I get what I agree to?

Once you get a notion in your head, it is impossible for change it. I must say that you are probably the most bigoted person I have ever encountered.

From my point of view, once you have decided I believe something, my denial of that belief has no effect of your decision.

The difference between 8 and 4 is 4. The difference between 27 and 23 is 4. The difference between 1,069 and 1,065 is 4. it's all the SAME DIFFERENCE, eh!?

Not relevant. As I pointed out in the post, it's not just the same difference, it's the same actual numbers.

So what does that prove? It's obvious, aint it? The numbers are all the same. They all give you the right answer. Therefore there can be no difference whatsoever between me having $1.069 (less 4) in the bank and me having $8 (less 4) in the bank.

What fool wouldn't understand that simple proof?

SR adherents understand it perfectly.


Yawn. If you can't understand the math, just acknowledge that. Failing that, find an actual error, rather than making inapt metaphors.

One Brow said...

Two guys, A & B, each exactly 6 feet tall, walk apart from each other for a certain distance. Then each one of them "appears" to be shorter.

They each get out their yardstick and "measure" how tall the other guy is at that distance. A determines that B is now only 5 feet tall. Guess what!?

B ALSO measures A to be only 5 feet tall!!!

All this while A and B both perceive themselves to be 6 feet tall!!

They get the exact same difference!


Exactly.

What can we conclude from this? Well, they're both only 5 feet tall, I guess. Or they're both 6 feet tall, it you prefer. Or, neither is 5 feet all and neither is 6 feet tall, because such things can't possibly be known, maybe.

We can conclude that valid direct comparisons need to ber sone side-by-side.

One thing for sure though: each one is taller than the other.

Due to the (metaphorical) relativity of height, the taller person is viewpoint dependent, until you bring them side-by-side.

"Two guys, A & B, each exactly 6 feet tall, walk apart from each other for a certain distance. Then each one of them "appears" to be shorter."

Yes, the changed appearance is "real." But, no, each has not actually ("really") shrunk. I'm sure you don't understand this distinction though. To you, they're both "real."


Got it. To you, they are both "real". However, I am wrong for thinking both are "real".

Somehow, though, in the HK experiment, each of the 3 clocks didn't manage to record less time elapsed than the other two. Go figure, eh?

Nor would SR predict that they would. Feel free to try to prove me wrong. Run that actual numbers, using SR as you interpret it. If you can produce the actual SR predictions, then do it. What are the numbers you keep talking about?

"Here, you are seeing a clock that has more or less time than some other clock, and you immediately jump to the conclusion that it must be something affecting the internals to the clock or its functioning.

There is no contradiction. The clocks read having passed through a different amount of time, and there was no change to the clocks themselves nor anything that interfered with their mechanisms."

Heh, are you aware of how clocks function? "simply passed through a different amount of time," eh? What the fuck does that mean? Time is a "thing" now, which you pass through, like a forest or something?


No, time is not a thing, anymore than distance it. Yes, a path in time is something you travel through/over, just like a path in space. If you drive to Kansas City, drive back, and park 10 feet from where the car started, it is not true that you only drove the car for ten feet. You can have distance along a path traveled; you can have time over a path traveled. The distance of a traveled path does not need to equal the shorest distance between two (spacetime) points; the interval of a traveled path does not need to equal the longest time between two (spacetime) points.

One Brow said...

Do you ACTUALLY think that clocks measure a thing called "time." A THING? Time is a concept, not a "thing," sorry.

So is distance, and there is a difference between the spatial distance a path uses and the distance between two objects. There is a difference between the temporal interval a path uses and the time between two events.

"Time" reaches into clocks and changes the position of the hands on the clock? Because, it is, after all, TIME?

You just can't see how ridiculous this assertion is, obviously.


I agree your assertion is ridiculous, and I do not hold to your assertion. Based on experience, this will not stop you from assigning this ridiculous position to me.

"SR allows you to use either the ECI or any individual TOV; regardless of choice, the satellite is slowed more. One flavor of LR says you have to choose the ECI, and then the satellite is slowed more. LR offers no advantage for this restriction."

Here again we revisit one of your most fundamental misconceptions. Nobody could convincingly claim to "understand" SR and even think about making such a claim.

You claim, with no basis whatsoever, that all frames of reference will predict the exact same amount of time dilation for all 3 clocks in the HK. Ridiculously wrong. This is the very claim that SR denies. But you think it is what SR claims.


It is what SR claims, according to all mainstream physicists. Yours is the outside position. You are the one claiming that all appendectomies are fatal, but that surgeons don't do appendectomies.

Any (and every) frame of reference which is moving with respect to another will, by SR postulates, have a different prediction about this matter. They will agree on the difference in speed, but they will, and MUST (in SR) disagree about which clock has slowed down.

As I painstaking detail above, in comparable situations, SR comes to the same numbers regardless of whom you assume is moving or not (as does LR).

If you disagree, point out the errors. DTFM (do the fucking math).

You seem to think that because they agree on relative speed, then they must agree about everything. The slightest amount of reflection would tell you otherwise, if you had even a cursory familiarity with SR, anyway.

The actual application of SR tells me they agree on everything. DTFM.

"You can't do LR without being in an inertial frame. In fact, you may have missed this, but every version of LR you presenting contains a discussion of the proper inertial frame for LR."

Of course you can. You can use LR to calculate the exact amount of time dilation he will experience both while he is blasting off (accelerating) and once he reaches "crusing" speed.


First you choose the inertial frame of the planet, then you can calculate the time dilation from that frame. Your point does not contradict my position.

Here's the main difference between LR and SR. In SR, the astronaut will claim he is not moving (when cruising). In LR he will acknowledge that he is moving, and will understand and acknowledge that HIS clock (not the earth's) has slowed down compared to his.

In SR, both the astronaut and the planet-side observer can choose either the cruising frame or the planet frame. Any numbers that can be directly compared will come out the same.

One Brow said...

I've said all this a thousand times. You still can't understand the difference between SR and LR.

They've sent chimpanzees into space. Maybe such as "astronaut" would think he is not moving toward the moon. But any human qualified to be an astronaut would NEVER think that. Only some confused SR advocate would claim that he does think that. Astronauts understand basic physics.


The math is easier when you assume the ECI is the inertial frame. That's the main difference.

"However, Jack's clock is 6 ls away at this point, so Jill sees -6 on Jack's clock. As Jill moves toward Jack, her clock goes from 0 to 8 while her view of Jack's moves from -6 to 10. She gains 8, she sees Jack's gain 16. Jill sees Jack's clock gains 2 to her clock gaining 1."

As Hogg (and any other decent physicists) makes clear, and as I have repeatedly pointed out to you, light speed delay has ABSOLUTELY NOTHING to do with time dilation or clock rates. Nor does the doppler effect. They are both totally irrelevant, and Hogg says you must CORRECT for this.


I agree, and nothing in my example indicates otherwise. I was just pointing out that, even with the assumption Jack is not moving, the observations are still symmetrical.

You think that, because they "see" it, that makes it relevant?

Makes what relevant? What is the "it" you think I am referring to?

Fraid not, but here again, you just demonstrate your total inability to distinguish between subjective and objective factors. You also seem to have great trouble distinguishing non sequiturs from pertinent, relevant considerations.

Relevant for what point?

"light speed delay has ABSOLUTELY NOTHING to do with time dilation or clock rates. Nor does the doppler effect."

Fowler just demonstrates his unconscionable sophistry by bringing this kinda crap into the discussion to confuse the issue and purport to "prove" his mistaken point.

You just demonstrate your susceptibility to accepting fallacious sophistry. No wonder you are so prone to use it in your own arguments.


I make no defense for Fowler's presentation.

aintnuthin said...

"I looked for some sign that you actually read the examples, looked at the numbers, and perhaps checked the calculations yourself. I did not see any such sign."

You're right. As I said, if someone tells me that the sum of 20 two-digit numbers adds up to a five-digit answer, I don't need to see what they add up to to know it's wrong.

You often do this. You posit some mistaken or irrelevant assumption and THEN start doing some mathematical operations, culminating in the smug suggestion that you have proved a point.

This goes way back to Jazzfanz days, when you kept insisting I look at some computer simulations of statistical odds. I didn't pay the least bit of attention to it then, because it was obviously completely irrelevan: a total non sequitur based on a conceptual misunderstanding of the issue then under investigation.

Math tells you NOTHING, it itself. It is only the assumptions/understanding which underly the math which are important. You have no clue about what is relevant or important here. I didn't analyze your calculations because they are irrelevant. You seem to think that the mechanical manipulation of numbers somehow provides you with "truth." Fraid not.

aintnuthin said...

"LR, every observer measures from their own viewpoint, and then uses the LT to correct those measurements to the preferred inertial frame."

Exactly. That's what I just said and have repeatedly said in the past. The issue is NOT what one "sees." The important thing is what's left after you have CORRECTED for the irrelevant and totally accidental idiosyncrasies of individual perception.

Being the solipsist that you are, you seem to think the uncorrected perceptions are what's relevant and importatn. They are merely incidental, and of no fundamental importance at all, as Hogg pointed out. You can't quite seem to grasp that.

aintnuthin said...

This post is something of an "aside," but not really. I got involved in that Jazzfanz plus/minus debate only because some poster claimed it was "scientific." There was no scientific methodology involved at all. No experimentation controlling variables, no attempt to establish a ceteris paribus standard, or anything remotely resembling it.

One could, of course, find some of the correlations "interesting," but it was grossly inappropriate to call any conclusions based on that data "scientific."

This is not at all atypical with some people. They are prone to pronounce their idle speculations as being "scientific" when they are no such thing.

And/or they regard "numbers" and mathematical calculations as somehow being inherently "scientific." They don't understand the scientific method at all, if that's their view.

aintnuthin said...

"n all three frames, at the comparison point C reads 34.64 and A reads 40, all of this regardless of whether A, B, or C is the one really at rest."

1. And? So? What do you think you have proven here, and how do you think it is relevant. I've already addressed the inherent flaw (for SR) in your conclusions. I just haven't taken the time to find and point just how and where you went wrong. You might want to do that for yourself.

2. The issue of "really at rest" is not even relevant, depending, perhaps, on what you mean by it. So why spend all that time and energy addressing it?

Consider what I said in another post: With respect to the solar system (not the whole universe) there IS one (and only one) point which is motionless with respect to object in the system.

In the HK experiment there is one, and ONLY one, frame of reference which accurately predicts the actual clock differences which were actually observed That, and that alone, is quite sufficient to treat it as a "preferred frame"---it gives the right answers. "Really moving," in some absolute sense, isn't even relevant, nor is such a concept required to employ SR.


aintnuthin said...

Edit: "... nor is such a concept required to employ SR." <--- I meant LR here, which should be obvious, but....

aintnuthin said...

You ask me how I think your analysis of SR is wrong. I have already done that countless times. What good will it do to repeat it?

Here's one example: You say:

"The fundamental premises specific to SR:

1. Identical experiments in inertial environments have identical results.
2. In particular, the speed of light in a vacuum moving inertially is always measured to be c.

As I have already pointed out, you have this backwards. LR, not SR, says that the speed of light is MEASURED TO BE the same in every frame of reference.

As self-contradictory as it is, SR does not say this. It says that the speed of light IS constant. Do you even understand the conceptual difference?

I asked you, in the last thread, to address the inherent contradiction here. How could it REALLY BE constant and yet ALSO be measured to be the same, if, as the LT presumes, measuring intruments are distorted?

You claimed there that you responded to this here. You haven't in the lease, other than you simply repeat your unexplained and illogical claim that "there is no contradiction." Obviously, you don't even understand the conceptual problem.

aintnuthin said...

"You can have distance along a path traveled; you can have time over a path traveled."

Of course. I've already addressed this many times. But a "longer time" is NOT equivalent to a "longer distance." To confuse and conflate the distinct notions of time and distance, and call that conflation a "path," doesn't change anything. Yes, an hour is "longer" (in terms of duration, not distance) than a half hour.

So what? Any child knows this. What does this simple truism tell you about the nature of the universe?

aintnuthin said...

Any mystical talk about "traveling through" spacetime is just self-deceptive hogwash. It is metaphorical, at best. There is no deep, profound conclusion to be drawn from mangling the concepts of space and time and saying they are somehow dependent upon each other and thereby form some mythical, non-existent "entity," notwithstanding Minkowski's self-delusion to the contrary.

Just another case where people let their religious devotion to math cloud their judgment and induce them to "believe" in self-contradictory premises.

You get to the mall faster if you travel at a higher speed. Big whoop. Like, who knew?

aintnuthin said...

"You dodged rather than answering. Is a solar eclipse a real event (one which does not change the nature of the sun, moon, nor earth)?"

You mean this assertion?:

"This time, can you come up with a better answer than 'a solar eclipse doesn't change the nature of the sun, moon, or earth'?"

I did "duck" the question at all, and addressed it at length. It's not even a question. Not one that makes any sense.

There is no "better answer." It's quite simple. You said it yourself but you can't see the significance because you're a solipsist.

The perspective from which you view an eclipse does NOT change the nature of the earth or the sun. What more is there to say? What the hell is the big mystery supposed to be here? Any kid can easily comprehend this, but you can't seem to.

aintnuthin said...

"https://en.wikipedia.org/wiki/Hafele%E2%80%93Keating_experiment
When reunited, the three sets of clocks were found to disagree with one another, and their differences were consistent with the predictions of special and general relativity"

Yeah, so? What does that mean? You do no thinking at all. What predictions? Your source says:



"According to special relativity, the rate of a clock is greatest according to an observer who is at rest with respect to the clock. In a frame of reference in which the clock is not at rest, the clock runs more slowly, as expressed by the Lorentz factor. This effect, called time dilation, has been confirmed in many tests of special relativity..."

The time dilation factors of Lorentz (which Al stole) are confirmed, get it? LR is confirmed, get it? The HK experiment did NOT use SR to explain the results.

I'll say it twice more, to bring the total to 102 from about 100:

The HK experiment did NOT use SR to explain the results

The HK experiment did NOT use SR to explain the results

You wouldn't know this, because you don't understand the basic suppositions of LR vs SR. By "understand," I mean you don't give any thought whatsoever to the basic, fundamental differences between the two. You just conflate one with the other.

aintnuthin said...

"https://en.wikipedia.org/wiki/Hafele%E2%80%93Keating_experiment
When reunited, the three sets of clocks were found to disagree with one another, and their differences were consistent with the predictions of special and general relativity.

You quote from this again. This is wrong. It only seems "right" to those who can't distinguish between SR and LR. "Mainstream" physicists know the difference. Using SR, you CANNOT choose, say, the naval observatory clock, as your "preferred frame," and get the result the clocks actually demonstrated. Nor can you get the correct answer by switching to the westbound clock. Or the eastbound clock.

Any simpleton would know that, but you don't. You glibly claim, with utter miscomprehension of SR, that every inertial from will give you the same exact clock readings that the clocks in question ACTUALLY showed. Your own elaboration concoction, design to "prove" your claim, DISPROVES it.

It just shows how woefully lost you are.

The HK results are not, and CAN NOT, be explained by referring to the "relative" motion between the clocks. They can only be explained by calculation how all three clocks are moving relative to the ECI (NOT each other).

aintnuthin said...

I've said this a hundred times, but you can never grasp it.

Let's use SR and look at things from "the perspective" of the Naval clock. It will predict that the EB an WB will both slow down, by equal amounts. This is NOT what the clocks actually showed.

Use the other frames, and you will get still different answers using SR, but they too will disagree with the actual observations.

Each viewpoint is NOT " equally valid." They will not all give you the "same prediction," either. There are, in fact, all equally INVALID when you look at the actual clock readings.

aintnuthin said...

"Each viewpoint is NOT " equally valid." They will not all give you the "same prediction," either. There are, in fact, all equally INVALID when you look at the actual clock readings."

A modicum of thought will affirm this point. I really don't care if you cite 10,000 wiki articles that say completely stupid bullshit, that will not induce me to abandon the inescapable logical conclusions. But it doesn't take 10,000 for you. Just one.

aintnuthin said...


"It is what SR claims, according to all mainstream physicists. Yours is the outside position. You are the one claiming that all appendectomies are fatal, but that surgeons don't do appendectomies.

Any (and every) frame of reference which is moving with respect to another will, by SR postulates, have a different prediction about this matter. They will agree on the difference in speed, but they will, and MUST (in SR) disagree about which clock has slowed down.

As I painstaking detail above, in comparable situations, SR comes to the same numbers regardless of whom you assume is moving or not (as does LR).

If you disagree, point out the errors. DTFM (do the fucking math).

You seem to think that because they agree on relative speed, then they must agree about everything. The slightest amount of reflection would tell you otherwise, if you had even a cursory familiarity with SR, anyway.

The actual application of SR tells me they agree on everything. DTFM."

=====

What a pitiful exchange. You don't need to do one bit of "math" to understand how wrong you are.

One of the first things you will be told about SR is that if A and B are moving with respect to each other, then:

1. A will see B's clock as slowing down, and

2. B will see A's clock as slowing down.

This is NOT the same answer. On the contrary, the two answers are polar opposites. You can't seem to understand that extremely simple point.

SR will go on the make the self-contradictory claim that "both are right," too. But that doesn't "explain things," It just creates an utterly absurdity for a sucker to "swallow." It is logically impossible for both to be right.

Eric, cognitive dissonance will NEVER be a problem for you. You can't think and accept conflicting thoughts (and then experience some "dissonanace"), because you don't think at all. You just parrot what you think the politically correct response should be. Pathetic, really.

aintnuthin said...

A: My clock reads 40, do you agree, B?

B: Yes, I agree that your clock reads 40.

BUZZ! BUZZ! BUZZ! WRONG ANSWER!

Wrong if you subscribe to SR, anyway. In SR, A and B will ever agree on what their clocks read. That's all you need to know. You don't need long-winded, badly articulated "explanations" which incorporate faulty assumptions and elaborate mathematical calculations to KNOW that, in SR, the two CAN NOT, and WILL NOT, agree on what their respective clocks read.

Take Fowler's example. Jack says his clock reads 10 and Jill's 8. Following SR, IF she were the one at rest, then her clock would read 10 and his 8. But they would NEVER agree that their respective clocks give the same readings for both of them. NEVER. It's not allowed in SR. In LR they would always agree, but that's LR, not SR.

aintnuthin said...

A: I have $1 right now, can we agree on that? B: Yeah, I agree.

A: OK, then, if I add $5 to it, I will have $1,000,000, right? B: Wrong, you will have $6.

====

It's OBVIOUS that A and B agree, isn't it. They agree that he has $1. So they are in COMPLETE AGREEMENT! Q.E.D.

aintnuthin said...

"In SR, both the astronaut and the planet-side observer can choose either the cruising frame or the planet frame."

This is completely wrong. Another fundamental misunderstanding of SR on your part. To help you, I quoted the Harvard physics prof, Morin, on this very point. As always, you are incapable of understanding what he says.

No, in SR, they are NOT "free" to choose any frame. In SR (but not LR) the astronaut MUST, repeat MUST, not may, treat himself as stationary. If he fails to do this, he is abandoning SR and resorting to LR. In Morin's word's, this would be a "complete disaster" for SR. Again, this is only about the 100th time I have pointed this out to you. You never understand a word, yet you think you know SR. It's sad, really.

aintnuthin said...

Question: If you are travelling at the rate of 80 mph, how long does it take you to go 80 miles?

To some the answer is immediately obvious. It requires no calculation, extensive reflection or thought, or anything even remotely like that. And the answer is indisputable. It is a matter of tautological "truth."

To others: 8 hours. No, wait, 10 minutes. No, let me change that--two hours. Actually, it's two minutes....ad infinitum.

Hard to explain the difference, but it definitely exists.

aintnuthin said...

Me: Yes, the changed appearance is "real." But, no, each has not actually ("really") shrunk. I'm sure you don't understand this distinction though. To you, they're both "real."

YOu: Got it. To you, they are both "real". However, I am wrong for thinking both are "real".

=====

You claim to believe in an "objective reality." But you don't. You are wrong because you cannot differentiate between subjective "reality" from objective "reality." For you, real is real, eh? You get completely lost with your own equivocating semantics.

No one claims that subjective sensations are not "real." You just impute that false position to others because YOU can't differentiate the subjective from the objective.

You say that OTHERS don't accept an "objective reality" if they don't take you solipsistic position that to be is to be perceived. But, of course, that position is the antithesis of one which accepts the existence of an objective reality.

aintnuthin said...

I have repeatedly cited mainstream physicists affirming that the speed of light is not constant in accelerating frames. This should be obvious even without citation to authorities. Simple reflection will tell you that, once you accept the premises underlying SR (and LR). Hence, SR cannot apply to any non-inertial frame of reference.

But what is really being said here?

If I'm sitting in my car, and then mash down on the gas pedal, have I suddenly changed the nature of the universe? Have I "caused" light to stop moving at a uniform speed. Have I changed the nature of light?

Think about it.

aintnuthin said...

Lightning strikes a cloud near me, causing a thunderclap sound. I hear the sound "now." A guy a mile away saw the lightning at (virtually) the same time I did. But he doesn't hear the sound until about 5 seconds later.

Question: Did the lightning strike and the thunder clap occur at two different times. One for the time I heard it, and another for the time he heard it?

A three year old might say "yes," no sound is created until you hear it. A "sophisticated" adult might say, yes, it's two different times, because "simultaneity is relative."

No reasonable scientist would ever say either, though. They would simple say that the lightning strike and the creation of the sound wave (the thunderclap) occurred at (virtually) the same time, regardless of subjective perception by different individuals.

What would you say?

aintnuthin said...


I said: "It's OBVIOUS that A and B agree, isn't it. They agree that he has $1. So they are in COMPLETE AGREEMENT! Q.E.D."

Often my responses to you are indirect, in the hope that I will stimulate you to actually think. That statement was a response to this one, of yours:

"The actual application of SR tells me they agree on everything. DTFM."

Do you see the connection? I doubt it, even if I make it explicit.

aintnuthin said...

"Any mystical talk about "traveling through" spacetime is just self-deceptive hogwash....You get to the mall faster if you travel at a higher speed. Big whoop. Like, who knew?"

Relativist: "Can't you see the obvious, fool!? That means the distance is shorter! Space shrinks! Time, not clocks, slows down!"

I'm not sure exactly how to continue this pseudo-religious rant, because I don't happen to subscribe to this particular brand of religious dogma. Since you do, maybe you can't articulate it all for me, eh?

aintnuthin said...

I will note that you have also said the following things, at one point or another:

"Time doesn't slow down. Time is not a physical object that can move more quickly or more slowly."

We completely agree here.

"Distance doesn't expand/shrink. Space is not a physical object that can be larger or smaller."

We completely agree on this point also.

So why do we disagree on anything?

Because you don't really believe in, consistently apply, and adhere to those statements, that's why. You just pay lip service to them, then immediately commence to ignore and contradict them.

You do NOT pay any attention to the logical implications of your own claims.

As a result, you end up making contradictory claims, yet say they are entirely consistent.

aintnuthin said...

In the wiki article on the twin paradox, this (among many other) claims is made:

"In their rest frame the distance between the Earth and the star system is εd = 0.6d = 2.4 light years (length contraction), for both the outward and return journeys."

Do you believe that, or do you believe, as you have claimed to, that:


"Distance doesn't expand/shrink. Space is not a physical object that can be larger or smaller."

Notice that the parenthetical phrase "length contraction," as used by wiki, is inaccurate. They are NOT talking about length contraction. "Length is the distance between the beginning and end point of a particular object, like, say, a yardstick. The distance (space) between two separate objects is the distance (not length).

Wiki immediately transmutes the length contraction of the LT into a metaphysical "distance contraction" wherein space "shrinks."

Which view do you subscribe to? Does space shrink, as wiki claims, or not?

You have conceded that "measurements" can be inaccurate at times. At other times, you act as though whatever is measured is what *is.*

Which is it? Are you capable of making any kind of distinction between what "is measured" and what "is?"

aintnuthin said...

Another way wiki could have put it would have been:

"Because it is NOT moving, the yardsticks on the spaceship are actually LONGER than those on earth. As a result of using longer yardsticks, they MEASURE the distance to be shorter. Of course the actual distance between the earth and the star remains unaltered, that distance is merely measured differently.

Because the earth IS moving, and the spaceship IS motionless, the earth mistakenly perceives the actual distance to be greater than it really is. But that is only because it is using distorted yardsticks."

Of course that would undermine the given premise, i.e., that the spaceship is actually travelling, and is NOT motionless. So which is it?

aintnuthin said...

Notice that wiki starts with this introductory phrase: ""In their rest frame"

"They" being the occupants of the travelling spaceship. But, if they are travelling, how can they be at "rest?"

Somebody is moving here, either the earth or the spaceship (or both). They cannot both be "at rest," even if they both think they are (or claim to be) "at rest."

So who is right? Who is moving, and who isn't? According to the standard resolution, the earth is the one who is "really" stationary. Therefore it is the travelling twin who will "really" age less.

This is just another way of saying that SR is in fact preposterous in it's self-contradictory claims (each is at rest) and that one must resort to an LR analysis wherein the earth is chosen as the preferred frame in order to arrive at the correct conclusion.

aintnuthin said...

Also notice that, from the viewpoint of the spaceship's "rest frame," as I said:


Because the earth IS moving, and the spaceship IS motionless, the earth mistakenly perceives the actual distance to be greater than it really is. But that is only because it is using distorted (shortened) yardsticks."

But notice: Now the earth has shorter yardsticks, not longer. But, by this hypothesis, it is also moving. But, if it is moving faster, then, according to the SR interpretation, the "distance" to the star would now be shorter, not longer.

This constant shifting back and forth between subjective viewpoints, treating each, in succession, as being "correct," just ends up in a never-ending series of inconsistencies. Not unexpected when you start out with a premise (both are at rest) that is logically inconsistent from jump street.

aintnuthin said...

What can we conclude from this? Well, they're both only 5 feet tall, I guess. Or they're both 6 feet tall, it you prefer. Or, neither is 5 feet all and neither is 6 feet tall, because such things can't possibly be known, maybe.

We can conclude that valid direct comparisons need to ber sone side-by-side.

One thing for sure though: each one is taller than the other.

Due to the (metaphorical) relativity of height, the taller person is viewpoint dependent, until you bring them side-by-side.

=====

Heh, and you claim to believe there is such a thing as "objective reality," eh? You are granting "reality" to the distorted perceptions of each observer here, that's all. You say "the taller person is viewpoint dependent." The viewpoint determines the facts, you say.

If "valid direct comparisons need to ber sone side-by-side," as you say, then all science is impossible. No two measurements made at different times, in different parts of the world, could ever be compared. If one guy measures the distance from Chicago to New Orleans to be 700 miles, no other guy can ever make a measurement which would constitute a "valid direct comparison." It wouldn't matter if he measured 700 miles, 699, 701, or whatever. You couldn't possibly compare the two, they were not made "side by side."

You have given the WRONG answer to this question, sorry. Hogg gives you the right answer. You must correct for differences which are due to inessential matters emanating from subjective perspective, and THEN compare. You don't abandon any and every attempt to compare measurements as being "invalid" if not taken "side by side."

You have, of course, advanced the standard evasion of the positivistic relativist and the creed he adheres to (while constantly violating in practice). It's' "meaningless."

aintnuthin said...

I said: hey each get out their yardstick and "measure" how tall the other guy is at that distance. A determines that B is now only 5 feet tall. Guess what!?

B ALSO measures A to be only 5 feet tall!!!

All this while A and B both perceive themselves to be 6 feet tall!!

They get the exact same difference!"

I was being facetious here, but you say:

"Exactly."

====

Exactly, what? I suspect you mean this proves they get the "same answer." But, as I have said. B says A is shorter than him. A says B is shorter than him.

This is NOT the same answer being given by both A and B. The "same difference" is not the "same answer."

aintnuthin said...


"There is no SR calculation where the sun clock runs slower than the earth. If you really believe otherwise, prove it. Run the actual numbers. Show me."

Heh, how can you even think that this is an open question and still claim to understand the underlying concepts of SR? This just shows how completely you confuse LR with SR.

i am on earth. I am in my own inertial "rest frame." I am going to employ SR to make time dilation calculations. With respect to me, the sun is moving (per SR, and the very definition of a "frame of reference, I am NOT moving, the sun is). I am adopting, as I must in SR, a "geocentric" view of the universe.

Since, from my frame, the sun is moving, then, per the LT, IT'S clocks have slowed down. Why? Because it is the one moving.

You really don't understand that this is what SR dictates?

aintnuthin said...

Several posts back I ended with this statement: "Take Fowler's example. Jack says his clock reads 10 and Jill's 8. Following SR, IF she were the one at rest, then her clock would read 10 and his 8. But they would NEVER agree that their respective clocks give the same readings for both of them. NEVER. It's not allowed in SR. In LR they would always agree, but that's LR, not SR."

Knowing you, you will say they did agree. They didn't. Sure they did (necessarily) agree about what they both saw at the same time and the same place. It should have ended there. Jill should have realized that SHE was the one moving, whether she previously knew it or not.

But Jill does NOT do that. In effect, she DENIES that Jack's clock reads 10. She says it "really" reads 6.4 ls, not 10. Of course this makes no sense, either, but don't blame me, blame Fowler. The two clocks are conceded to be synchronized in Jack's frame. One can't read 6.4 and the other 10. But of course Fowler's acts like he has made sense out of, and has "explained," why two clocks, when synchronized, can have different readings.

This is just another "If you're going 89 mph, how long does it take you to go 80 miles?" kind of question:

Q: If two clocks are synchronized, and one reads 10, what does the other read?

A: (from Jill, the buffoon) 6.4!!

Really, Jill? Why do you say that? SR makes me say it, that's why.

aintnuthin said...

That last post was just an elaboration on the preceding one which it referred to. I am trying to preempt what I think your misconception will be.

The main point is the same: In SR, two observers who are moving relative to each other will NEVER (per the dictates of SR, as a "theory") agree on what the other's clock reads.

You think otherwise, but only because you do not understand even the most rudimentary principles of the theory.

No math required.

aintnuthin said...

Jack and Jill will agree on one thing: Her clock reads 8. But this is only because she is the one moving, which she denies.

But this doesn't mean they "completely agree on everything" as you assert. Jack will say (know) that the first clock also reads 10, because the two are synchronized in his frame. Jill will claim it reads 6.4 (which is the equivalent of saying the one she is looking at, which reads 10, "really" reads 6.4).

Why? Because she is a denialist who CANNOT and WILL NOT be deterred from her assertions by either facts or logic. In SR she MUST deny that she is moving (which would mean that, if her clock reads 8, then Jack's clock *HAS* to read 6.4, irrespective of what she sees).

I already know that it is extremely unlikely that you will be able to comprehend this simple demonstration.

aintnuthin said...

Some people seem to think they are saying something very profound if they say "simultaneity is relative."

They are not. They also like to act like they know exactly what they are talking about when they use that phrase, when they don't. I explained, in very simple terms, the difference between relative and absolute simultaneity.

Your only comment, as I recall, was to say that you didn't agree with my interpretation. Of course you didn't say what *your* interpretation was, and I don't think you would be able to give an alternative interpretation. If you did, it would be wrong.

I am not talking about "my" interpretation to begin with. I am talking about Einstein's interpretation, as explained in his own words, using his own illustrative examples.

Al's example is a lot like Fowler's, and just as fallacious.

1. Al posit that one guy (on a train) is moving and another, by the tracks, is stationary relative to the earth's surface. OK, now what

2. He uses his knowledge of their motion to explain why the light from two different lightning strikes reach the eyes of the two observer's at different times.

OK, fair enough, an quite easy to understand. This is just a case where you should correct for the perspectival difference created by then motion, and then compare.

The guy on the train should, when calculating whether the two strikes were simultaneous, need only (and should) calculate the consequences of his motion and factor that out. If he does that HE, like the guy by the tracks, will say that the two strikes WERE simultaneous.

End of problem. Very simple actually. Al has told us who is moving. After factoring this in they AGREE that the strikes were simultaneous.

One problem. To get his fancy-ass notion of "relative simultaneity" he has to prohibit this simple correction. How does he do that. Easy, even though HE knows the guy on the train is moving, he prohibits the guy on the train from acknowleding that. He must FALSELY insist that he is stationary while everything attached to the earth's surface whizzes past him. Al KNOWS this is false, but he nonetheless forces the train passenger to adopt this mistaken claim.

As I have said (as as Morin said), Al must insert a fool into the situation who doesn't know that he boarded a train and, for that reason, is now moving relative to the earth. Only by creating a chump, who is prohibited from knowing what Al knows, can he claim that, "simultaneity is relative." It aint. The fact that the train passenger, being a fool, "sees" the two as non-simultaneous is irrelevant. He is mistaken in what he thinks explains what he sees."

You might just as well say that if I see a supernova explosion, which occurred long ago, and far away, now then it happens NOW, for me. And, if it happened now, for me, then, BY GOD, it happened NOW, not before. I am too ignorant to understand that it COULDN'T have happened now, if I see it now.

Only an idiot would think that what he "sees" is, by necessity, what IS TRUE. Or, just replace "idiot" with "solipsist." Same thing, in my book



aintnuthin said...

You might just as well say that if I see a supernova explosion, which occurred long ago, and far away, now then it happens NOW, for me. And, if it happened now, for me, then, BY GOD, it happened NOW, not before. I am too ignorant to understand that it COULDN'T have happened now, if I see it now.

Only an idiot would think that what he "sees" is, by necessity, what IS TRUE. Or, just replace "idiot" with "solipsist." Same thing, in my book.

====

Let's say the star was 10 million light years away when it went supernova. A guy who was there at the time would say it happened 10 million years ago. Now suppose I'm NOT an idiot who thinks everything is all about me and that it could have only happened at the very instant I saw it. If I say that, as a solipist, then I am saying, quite profoundly, no doubt, that "simultaneity is relative").

So what do I say if I'm not an idiot? I say to the guy who was there: "I agree with you. This event occurred 10 million years ago."

What's that called, if we agree? It's called "absolute simultaneity."

aintnuthin said...

Anyone who claims that Einstein "proved" that "simultaneity is relative" has about as much sense of what "proof," and "science," is as someone who claims that whatever conclusions he wants to draw from a compilation of plus/minus data in a basketball game are "scientific."

aintnuthin said...

So, you ask, what practical difference does it make if you use SR (which posits that simultaneity is relative) or LR (which posits absolute simultaneity))?

I just gave you one "practical difference." If simultaneity is relative, then I say any supernova I see, anytime, "happened" at the exact moment I saw it. This is a rather naive view of "science," eh? It in fact PROHIBITS science. As Al noted, physics MUST presuppose the existence of an "objective reality." But the view taken here DENIES that there is any objective reality. The only "reality" that exists in this view is strictly subjective. It is, as Al also noted, solipsism, which precludes science (because physics must presume the existence of an objective reality, in addition to a "subjective reality").

aintnuthin said...

All said and done, Eric, it's not very complicated. If you could ever grasp the following two points, you might be on the way to understanding SR, LR, and the difference between the two:

1. What you "measure" something to be (time and distance for example) is not necessarily "true." If your measuring instruments are distorted (by heat, cold, motion, whatever) then you will not get the "true" (as established by the accepted etalons) dimensions of what you're trying to accurately measure (time or distance).

2. Subjective reality is different than objective reality. The two will not necessarily lead you to the same conclusions. Subject reality is NOT the equivalent of, and identical to, objective reality.

One Brow said...

A summary of some points below.

1) The first principle of relativity (identical experiments in different, inertial frames produce identical results) sets a limitation on the types of movement SR refers to when it claims, on a result of experimentation/measurement-within-your-frame, that you can't tell who is moving. SR is saying that you can't tell which inertial environment is the one not moving. You can run experiments to determine acceleration, so you can tell that accelerated environments have moved in SR. You are also allowed to determine who is moving by referring to additional principles that are not the results of experimentation/measurement-within-your-frame.

2) You assume that Jill sees Jack's clock as being synchronized with clock1. Based on what Jill actually sees when she passes clock1, this is impossible. Using the fact that she sees the -6 on Jack’s clock as clock1 passes her, that she measures the distance between clock1 and Jack to be 4.8ls, and that the relative motion between her and Jack is .6c, the only possible number to actually be on Jack’s clock is 3.4; this calculation by Jill does not use the LT.

3) It may have been hyperbole, but towards the end, you conflated relativity of simultaneity with the notion that events happen when you experience them, but the relativity of simultaneity actually denies this is true.

4) I still don't know what to make of your ontology. You seem to deny any sort of reality to anything not strictly material, yet claim some of these purely conceptual things must be invariant from person to person. I don't believe in invariant mental concepts; I do accept non-material, physical notions like distance and physical perspective.

"I looked for some sign that you actually read the examples, looked at the numbers, and perhaps checked the calculations yourself. I did not see any such sign."

You're right. As I said, if someone tells me that the sum of 20 two-digit numbers adds up to a five-digit answer, I don't need to see what they add up to to know it's wrong.


OK. I didn't make any similar claim, though.

You often do this. You posit some mistaken or irrelevant assumption and THEN start doing some mathematical operations, culminating in the smug suggestion that you have proved a point.

By contrast, you find some obscure people advocating for some minority viewpoint, often based on completely misunderstanding the topic and hand, and then bray loudly about how 99% of scientists are misinformed and ignorant.

This goes way back to Jazzfanz days, when you kept insisting I look at some computer simulations of statistical odds. I didn't pay the least bit of attention to it then, because it was obviously completely irrelevan: a total non sequitur based on a conceptual misunderstanding of the issue then under investigation.

There was a conceptual error on my part. Also, there was a complete inability on your part to explain the error, because you don't dive deeply enough into subjects to genuinely understand and capably interpret them, but rely on surface impressions and the opinions of others. If you want to demonstrate I am making the same error, you’ll need to explain why I am misstating or misinterpreting the first principle of SR. Making unsupported claims, over and over, is not making such a demonstration.

Math tells you NOTHING, it itself. It is only the assumptions/understanding which underly the math which are important.

You are now making the opposite error I did. The underlying assumptions are important, but they can only be judged by their ability to make successful predictions, and to make those predictions, you have to DTFM.

I learned from my error.

One Brow said...

You have no clue about what is relevant or important here. I didn't analyze your calculations because they are irrelevant. You seem to think that the mechanical manipulation of numbers somehow provides you with "truth." Fraid not.

I don't know about "truth". All I'm looking for are understanding and implications. You can only get very shallow understanding in either category if you don't DTFM.

"LR, every observer measures from their own viewpoint, and then uses the LT to correct those measurements to the preferred inertial frame."

Exactly. That's what I just said and have repeatedly said in the past. The issue is NOT what one "sees." The important thing is what's left after you have CORRECTED for the irrelevant and totally accidental idiosyncrasies of individual perception.


Thank you for agreeing that I understand LR.

Being the solipsist that you are, you seem to think the uncorrected perceptions are what's relevant and importatn. They are merely incidental, and of no fundamental importance at all, as Hogg pointed out. You can't quite seem to grasp that.

Being a shallow and ill-educated student of relativity (not implying you are this as a person overall), you fail to see that by talking about measurements of the world as a reflection of reality, I'm advocating for the opposite of solipsism. If the observations a moving observer makes had no correspondence to reality, the LT could not make them relevant. It is because these observations reflect reality that the LT can work.

This post is something of an "aside," but not really. I got involved in that Jazzfanz plus/minus debate only because some poster claimed it was "scientific." There was no scientific methodology involved at all. No experimentation controlling variables, no attempt to establish a ceteris paribus standard, or anything remotely resembling it.

I agree there were no double-blind experiments or other attempts to control other variables. Not all science is experimental, some is based on the study of populations. You can certainly these studies are open to methodological problems, that correlation is not causation, etc. However, "not scientific" may be too strong.

One could, of course, find some of the correlations "interesting," but it was grossly inappropriate to call any conclusions based on that data "scientific."

This is not at all atypical with some people. They are prone to pronounce their idle speculations as being "scientific" when they are no such thing.

And/or they regard "numbers" and mathematical calculations as somehow being inherently "scientific." They don't understand the scientific method at all, if that's their view.


Is it scientific to say there is no connection between x and y by studying various populations? In many cases, there have been no controlled double-blind studies for ethical concerns. If not scientific, what type of knowledge is a collection of studies that conclude a lack of correlation between x and y?

For +/- specifically, to my recollection there are such large error bars that using it for any individual game is meaningless.

"In all three frames, at the comparison point C reads 34.64 and A reads 40, all of this regardless of whether A, B, or C is the one really at rest."

1. And? So? What do you think you have proven here, and how do you think it is relevant.


I showed that, regardless of whether A, B, or C is really at rest, the comparable numbers always come out the same; so you can't, by comparing clocks, say who is really moving.

One Brow said...

I've already addressed the inherent flaw (for SR) in your conclusions.

Which principle of SR does my example violate? If none, expand on how this shows a flaw for SR.

I just haven't taken the time to find and point just how and where you went wrong. You might want to do that for yourself.

It's certainly possible I went wrong, but I have gone over these scenarios many times. I revised them several times after finding errors. If there are more errors to be found, it is unlikely I will find them.

2. The issue of "really at rest" is not even relevant, depending, perhaps, on what you mean by it. So why spend all that time and energy addressing it?

As you agreed above, the whole point of LR is to establish who is really at rest (more accurately, which frame is the inertial rest frame, as opposed to the inertial frames that are not rest frames). I am spending time and energy to point out that the choice of rest frame is irrelevant. That is the first principle of SR.

Consider what I said in another post: With respect to the solar system (not the whole universe) there IS one (and only one) point which is motionless with respect to object in the system.

In the HK experiment there is one, and ONLY one, frame of reference which accurately predicts the actual clock differences which were actually observed That, and that alone, is quite sufficient to treat it as a "preferred frame"---it gives the right answers.


See, this is part of the problem of not being able to DTFM; you offer opinions contrary to the results of the actual calculations. You can use the ECI in the HK experiments to get the right answer. You can use the SCI (the math is harder, but it can be done). You can use the CMB. These are three different frames, all effectively inertial for the purposes of HK, and they all give the exact same answers for the times shown on the planes and on the earth-bound clock. Not just the same differences, the EXACT SAME NUMBERS.

You can go on and on about how that's not possible, but you will be doing so without understanding the problem in any meaningful way. It's not only possible, it's mandated by the LT.

"Really moving," in some absolute sense, isn't even relevant, nor is such a concept required to employ LR.(sic)

I have acknowledged this many times. Feel free to keep repeating it.

You ask me how I think your analysis of SR is wrong. I have already done that countless times. What good will it do to repeat it?

Here's one example: You say:

"The fundamental premises specific to SR:

1. Identical experiments in inertial environments have identical results.
2. In particular, the speed of light in a vacuum moving inertially is always measured to be c.

As I have already pointed out, you have this backwards. LR, not SR, says that the speed of light is MEASURED TO BE the same in every frame of reference.

As self-contradictory as it is, SR does not say this. It says that the speed of light IS constant. Do you even understand the conceptual difference?


You are proposing that there is some sort of notion of speed in SR which is not the speed that is measured, but a pure speed may be different from the measured speed? Where do you get this from?

SR is a scientific theory; a theory about what we see, observe, and measure, as scientific theories are. If you measure light in a vacuum, without making an error, from any inertial frame, the light moves at c. Among other things, this is a consequence of Maxwell's equations. In SR, speed measured from any inertial speed is the true speed in that frame, and can be converted to other frames.

One Brow said...

I asked you, in the last thread, to address the inherent contradiction here. How could it REALLY BE constant and yet ALSO be measured to be the same, if, as the LT presumes, measuring intruments are distorted?

You claimed there that you responded to this here. You haven't in the lease, other than you simply repeat your unexplained and illogical claim that "there is no contradiction." Obviously, you don't even understand the conceptual problem.


I pointed out it was comment 7, I will quote what I said earlier (correcting some spelling).
The distortions are by the same factor (let’s call it g). Create two posts Y and Z in the same inertial frame with distance x. It takes light time t to move between them. The light moves at x/t. In another frame, the length of the posts is x', and the amount of time is t'. Since x' = xg, and t' = tg, you get x'/t' = (xg)/(tg) = x/t. The speed of light comes out the same.

It measures the same because the distortions (as LR interprets it) cancel each other out. This effect is predicted in any version of LR as well as SR.

"You can have distance along a path traveled; you can have time over a path traveled."

Of course. I've already addressed this many times. But a "longer time" is NOT equivalent to a "longer distance."


Agreed.

To confuse and conflate the distinct notions of time and distance, and call that conflation a "path," doesn't change anything. Yes, an hour is "longer" (in terms of duration, not distance) than a half hour.

Agreed, but not relevant.

So what? Any child knows this. What does this simple truism tell you about the nature of the universe?

Very little. However, it's not relevant to my explanation.

Objects take a path through time, the same way they do through space. Clocks measure the time, much as rulers measure distance. If event A and event B happen an hour apart, and a clock is ticking in between, the clock does not suddenly move forward one hour to show the difference between the beginning and the ending of the interval; it moves continuously on a path through time. Just like when you move from spatial point M to spatial point N, you don't teleport; you move continuously.

This continuous movement means that, just as you can take different paths of different path lengths in space to move between M and N, you can take different paths of different path lengths in time to move between event A and event B. Not because time is reaching out to change the clocks; the road does not reach out to change your odometer on your car. The path you take changes what the instrument will measure.

You might feel this is mystical. I'm OK with that. I see it as less mystical than some force that is physically distorting rods and clocks at speed, yet has no other physical properties.

Any mystical talk about "traveling through" spacetime is just self-deceptive hogwash. It is metaphorical, at best. There is no deep, profound conclusion to be drawn from mangling the concepts of space and time and saying they are somehow dependent upon each other and thereby form some mythical, non-existent "entity," notwithstanding Minkowski's self-delusion to the contrary.

You can choose your own religion here. At least mine doesn't require an undetectable, physical presence that compresses all measuring devices the exact same way and changes all clocks the exact same way.

Just another case where people let their religious devotion to math cloud their judgment and induce them to "believe" in self-contradictory premises.

There's a big difference between mystical and self-contradictory.

One Brow said...

You get to the mall faster if you travel at a higher speed. Big whoop. Like, who knew?

Again, not relevant.

"You dodged rather than answering. Is a solar eclipse a real event (one which does not change the nature of the sun, moon, nor earth)?"

You mean this assertion?:

"This time, can you come up with a better answer than 'a solar eclipse doesn't change the nature of the sun, moon, or earth'?"

I did "duck" the question at all, and addressed it at length. It's not even a question. Not one that makes any sense.


Solar eclipses have driven religious fervor and scientific studies of the sun's corona. It's one thing to say they are not physical, we could then discuss what category they belong to. It's another to say you can't even sensibly ask the question. If a solar eclipse is not real, how are scientists able to predict their occurrence? If I say there will be a solar eclipse 2016-11-12 at 1pm CST over Belleville, is there any way to verify if I am right or wrong? How can you be right or wrong about an event when you can't even ask about whether it is real or not?

There is no "better answer." It's quite simple. You said it yourself but you can't see the significance because you're a solipsist.

You're the one pretending that solar eclipses are such fancies of flight that their reality can't even be discussed. I don't believe you accept that you can't verify if the moon is keeping light from the sun from arriving at point X on the earth.

Your ontology seems seriously broken to me. I was trying to discuss it earlier, because it keeps interfering in our discussion of relativity. You deny the existence of the easily observable, and put forth the existence of the unverifiable. Yet, you don't explain any rationale for this holding together. I think this is the real gap between our understandings of relativity.

The perspective from which you view an eclipse does NOT change the nature of the earth or the sun. What more is there to say?

This much more: you are at point X. The moon is the only thing between you and the sun. You can't see the sun.

As opposed to: you are on point X. There is nothing between you and the sun. You can see the sun.

We agree that there is no difference in the sun, moon or earth. Are these scenarios therefore completely identical in every physical manner?

What the hell is the big mystery supposed to be here? Any kid can easily comprehend this, but you can't seem to.

I can't comprehend what I am outright stating and affirming? Possible, I suppose, but you will have to do much more to demonstrate that. However, that still leaves the question above unanswered. Are the two situations identical in every physical aspect?

"https://en.wikipedia.org/wiki/Hafele%E2%80%93Keating_experiment
When reunited, the three sets of clocks were found to disagree with one another, and their differences were consistent with the predictions of special and general relativity"

Yeah, so? What does that mean? You do no thinking at all. What predictions? Your source says:

"According to special relativity, the rate of a clock is greatest according to an observer who is at rest with respect to the clock. In a frame of reference in which the clock is not at rest, the clock runs more slowly, as expressed by the Lorentz factor. This effect, called time dilation, has been confirmed in many tests of special relativity..."

The time dilation factors of Lorentz (which Al stole) are confirmed, get it? LR is confirmed, get it? The HK experiment did NOT use SR to explain the results.


LR is SR + pre-chosen rest frame, get it? You can't use LR without using SR, get it? According to all of the mainstream physicists, the HK experiment did use SR. What makes you so much more knowledgeable?

One Brow said...

I'll say it twice more, to bring the total to 102 from about 100:

The HK experiment did NOT use SR to explain the results

The HK experiment did NOT use SR to explain the results


I'm happy to repeat, as well.

You can't use LR without using SR.

You can't use LR without using SR.

You wouldn't know this, because you don't understand the basic suppositions of LR vs SR. By "understand," I mean you don't give any thought whatsoever to the basic, fundamental differences between the two. You just conflate one with the other.

Coming from a person who can't even accurately state the premises of SR, I'm not impressed by your claim.

"https://en.wikipedia.org/wiki/Hafele%E2%80%93Keating_experiment
When reunited, the three sets of clocks were found to disagree with one another, and their differences were consistent with the predictions of special and general relativity.

You quote from this again. This is wrong.


Incorrect. It is accurate. What is wrong is your understanding of SR.

It only seems "right" to those who can't distinguish between SR and LR. "Mainstream" physicists know the difference. Using SR, you CANNOT choose, say, the naval observatory clock, as your "preferred frame," and get the result the clocks actually demonstrated. Nor can you get the correct answer by switching to the westbound clock. Or the eastbound clock.

That is correct. None among the naval observatory clock, the west-bound clock, and the east-bound clock are in an inertial frame; they can't be a preferred inertial frame because they are not an inertial frame at all. Therefore, none can be calculated as the preferred inertial frame. You can use the ECI, SCI, or CMB, as three examples of an infinite number of inertial frames. You can use any one of those infinite number of inertial frames. None of those infinite frames available are the frames of any of the clocks in the HK experiment.

Any simpleton would know that, but you don't. You glibly claim, with utter miscomprehension of SR, that every inertial from will give you the same exact clock readings that the clocks in question ACTUALLY showed.

Whether my claim shows comprehension or not, it is true. It is a feature of the LT.

Your own elaboration concoction, design to "prove" your claim, DISPROVES it.

Your failure to DTFM, or even RTFM, means that you can't even address what my claim does or does not prove.

It just shows how woefully lost you are.

It's kind of sad, really, watching you posture and puff in your ignorance. A sort of Dunning-Kruger archetype.

The HK results are not, and CAN NOT, be explained by referring to the "relative" motion between the clocks. They can only be explained by calculation how all three clocks are moving relative to the ECI (NOT each other).

They can be explained relative to the ECI, SCI, GCI (galaxy), CMB, any OTV of any satellite, or any other inertial frame. Whether you choose the ECI, SCI, or any other inertial frame, the predictions for the numbers (not just the differences, the actual numbers) on the clocks is identical.

I've said this a hundred times, but you can never grasp it.

I grasp what you are saying. It’s neither complex nor deep. You're just wrong.

One Brow said...

Let's use SR and look at things from "the perspective" of the Naval clock.

The naval clock is not in an inertial frame. You can do experiments to demonstrate this. For example, a Foucault's pendulum or a test for a Coriolis affect.

Again, the first principle of SR is 'Identical experiments in inertial environments have identical results'. The whole notion of "you can't tell you are moving" is based in this principle. However, you can perform experiments in the naval base that get different results from the same experiment in an inertial environment. The naval base fails to meet the first principle for SR.

It will predict that the EB an WB will both slow down, by equal amounts. This is NOT what the clocks actually showed.

The naval base fails to meet the first principle for SR, so SR makes no prediction from its viewpoint.

Use the other frames, and you will get still different answers using SR, but they too will disagree with the actual observations.

If you use an inertial frame, as SR requires, you always get back the same answer for the HK experiment. There are an infinite number of inertial frames you can choose from.

Each viewpoint is NOT " equally valid."

Agreed. Every *inertial* viewpoint is equally valid, but non-inertial viewpoints are usually invalid.

They will not all give you the "same prediction," either. There are, in fact, all equally INVALID when you look at the actual clock readings.

Agreed. Every inertial viewpoint gives you the same prediction, but attempting to use only SR from non-inertial viewpoints will give you the wrong predictions.

A modicum of thought will affirm this point. I really don't care if you cite 10,000 wiki articles that say completely stupid bullshit, that will not induce me to abandon the inescapable logical conclusions. But it doesn't take 10,000 for you. Just one.

Your logical conclusion is built on a false premise of SR treating non-inertial viewpoints as valid. It has no value.

"It is what SR claims, according to all mainstream physicists. Yours is the outside position. You are the one claiming that all appendectomies are fatal, but that surgeons don't do appendectomies. "

Any (and every) frame of reference which is moving with respect to another will, by SR postulates, have a different prediction about this matter. They will agree on the difference in speed, but they will, and MUST (in SR) disagree about which clock has slowed down.


When you DTFM, it always turns out that, for any numbers that can be directly compared (that is, appear in the exact same place at the same time, or close enough to make relativistic effects too small to make a difference), SR makes the same predictions from any inertial frame. This is not some coincidence, it is a part of the structure of the LT themselves.

What a pitiful exchange. You don't need to do one bit of "math" to understand how wrong you are.

That's the same behavior you reproach me for.

One of the first things you will be told about SR is that if A and B are moving with respect to each other, then:

1. A will see B's clock as slowing down, and

2. B will see A's clock as slowing down.

This is NOT the same answer. On the contrary, the two answers are polar opposites.


I agree. However, if A and B are in inertial frames, there will be at most one occasion where they can directly compare their clock numbers. At any time before/after that, A compares his actual clock number to his measurement of B's clock, and B compares his actual clock number to his measurement of A's clock, but the clocks are only directly compared once, at most. So, A can not prove B wrong, and B can not prove A wrong.

One Brow said...

You can't seem to understand that extremely simple point.

I understand the point. I don't see a real-world relevance for it.

SR will go on the make the self-contradictory claim that "both are right," too. But that doesn't "explain things," It just creates an utterly absurdity for a sucker to "swallow." It is logically impossible for both to be right.

Who is right, or wrong, depends on choosing a true (not necessarily absolute) rest frame. No observation made by A nor by B can do that. So, it's not that both are right, but rather, that there is no reason to be found to declare one is right and the other wrong from the measurements themselves. You have to use other means.

Eric, cognitive dissonance will NEVER be a problem for you. You can't think and accept conflicting thoughts (and then experience some "dissonanace"), because you don't think at all. You just parrot what you think the politically correct response should be. Pathetic, really.

Since I have expanded upon notion like path length, and the scenarios listed above, in ways I have never read, I believe I have sufficient evidence that I am doing more than parroting.

A: My clock reads 40, do you agree, B?

B: Yes, I agree that your clock reads 40.

BUZZ! BUZZ! BUZZ! WRONG ANSWER!

Wrong if you subscribe to SR, anyway. In SR, A and B will ever(sic, I think you mean never) agree on what their clocks read. That's all you need to know. You don't need long-winded, badly articulated "explanations" which incorporate faulty assumptions and elaborate mathematical calculations to KNOW that, in SR, the two CAN NOT, and WILL NOT, agree on what their respective clocks read.


They will agree at the one point where the clocks pass each other.

Take Fowler's example. Jack says his clock reads 10 and Jill's 8. Following SR, IF she were the one at rest, then her clock would read 10 and his 8. But they would NEVER agree that their respective clocks give the same readings for both of them. NEVER. It's not allowed in SR. In LR they would always agree, but that's LR, not SR.

If you like, I can lay out the math for Fowler's example. The short version: Jack and Jill agree on how both clocks read when Jill passes Jack. Jack's clock and clock1 are synchronized in Jack's frame, but not Jill's. In Jill's frame, Jack's clock starts out reading 3.6s, and has another 6.4 pass, so it reads 10 when Jill's clock passes (reading 8).

Now, you'll probably say that Jack's clock can't read 0 and 3.6 at the same time. I agree. However, Jack is many light-seconds away from Jill when she passes clock1, and neither Jill nor a clock1 observer can directly verify what Jack's reads. If you say they can look at Jack's clock and see, and that they should see the same number, you are again correct. They can both look right then at jack's clock, and see right then the number -6. For clock1 to see -6, that means Jack's clock currently must read 0 in clock1’s frame. For Jill to see -6, Jack's clock must currently read 3.6 in Jill's frame.

A: I have $1 right now, can we agree on that? B: Yeah, I agree.

A: OK, then, if I add $5 to it, I will have $1,000,000, right? B: Wrong, you will have $6.

====

It's OBVIOUS that A and B agree, isn't it. They agree that he has $1. So they are in COMPLETE AGREEMENT! Q.E.D.


Yawn.

One Brow said...

"In SR, both the astronaut and the planet-side observer can choose either the cruising frame or the planet frame."

This is completely wrong. Another fundamental misunderstanding of SR on your part. To help you, I quoted the Harvard physics prof, Morin, on this very point. As always, you are incapable of understanding what he says.


Morin says: If both A and B are blindfolded, they can still tell who is doing the traveling, because B will feel the acceleration at the turnaround.

The planet frame has no acceleration from the turn-around; you can apply SR from this viewpoint. The cruising frame (outbound) has no acceleration from the turn-around; you can apply SR from this viewpoint. The cruising frame (inbound) has no acceleration from the turn-around; you can apply SR from this viewpoint. The entire astronaut frame has acceleration; you can't apply SR from this viewpoint.

The first principle of SR is 'Identical experiments in inertial environments have identical results'. The whole notion of "you can't tell you are moving" is based in this principle. The astronaut knows he is moving from the acceleration, SR does not apply to his frame from the entire journey.

No, in SR, they are NOT "free" to choose any frame. In SR (but not LR) the astronaut MUST, repeat MUST, not may, treat himself as stationary. If he fails to do this, he is abandoning SR and resorting to LR.

Actually, this is just the opposite. The astronaut, experiencing an acceleration that violates the first principle of using SR, is not allowed to use SR (only) from his point of view.

In Morin's word's, this would be a "complete disaster" for SR. Again, this is only about the 100th time I have pointed this out to you. You never understand a word, yet you think you know SR. It's sad, really.

In Morin's words, that you quoted, the astronaut experiences acceleration, and therefore knows he is moving. When you quote Morin as saying the astronaut is definitively moving within an SR framework, you should be unsurprised that he is not claiming that the astronaut can claim he is not moving.

Question: If you are travelling at the rate of 80 mph, how long does it take you to go 80 miles?

To some the answer is immediately obvious. It requires no calculation, extensive reflection or thought, or anything even remotely like that. And the answer is indisputable. It is a matter of tautological "truth."


Yep. Another matter of tautological truth is that theories only apply in the situations that they set up to handle. SR's first principle means it only applies to calculations from inertial frames.

To others: 8 hours. No, wait, 10 minutes. No, let me change that--two hours. Actually, it's two minutes....ad infinitum.

Hard to explain the difference, but it definitely exists.


Yep.

One Brow said...

Me: Yes, the changed appearance is "real." But, no, each has not actually ("really") shrunk. I'm sure you don't understand this distinction though. To you, they're both "real."

YOu: Got it. To you, they are both "real". However, I am wrong for thinking both are "real".

=====

You claim to believe in an "objective reality." But you don't. You are wrong because you cannot differentiate between subjective "reality" from objective "reality." For you, real is real, eh? You get completely lost with your own equivocating semantics.

No one claims that subjective sensations are not "real." You just impute that false position to others because YOU can't differentiate the subjective from the objective.


You can't even use the word real without putting it in scare quotes. I believe that our subjective reality is an interpretation and a reflection of an underlying objective reality. I also believe that physical positioning is a matter of objective reality. A solar eclipse over Belleville is an objectively verifiable event. It is real, no scare-quotes.

The word "perspective" can refer to the effects of physical position, or the mental evaluation of phenomena. While it refers to similar concepts, these two applications have different states of reality. Perspective from physical position (perspective-p) is objectively real. Perspective from mental evaluation (perspective-m) has no physical reality.

Your primary argument against perspective-p is that your perspective-p regarding object X does not alter the nature of X. I agree that the nature of X is not altered, but find this to be an invalid argument against the reality of perspective-p.

You say that OTHERS don't accept an "objective reality" if they don't take you solipsistic position that to be is to be perceived. But, of course, that position is the antithesis of one which accepts the existence of an objective reality.

Among several recent examples, I stated explicitly that the goalposts, and the distance between them, do not change when there is no one to observe them. Your claim, that distance is a mental concept, is the one taking a physical distance and removing some of the hard reality from it. For me, distance is a physical phenomenon. I accept more physical reality than you, not less.

I have repeatedly cited mainstream physicists affirming that the speed of light is not constant in accelerating frames. This should be obvious even without citation to authorities. Simple reflection will tell you that, once you accept the premises underlying SR (and LR). Hence, SR cannot apply to any non-inertial frame of reference.

But what is really being said here?

If I'm sitting in my car, and then mash down on the gas pedal, have I suddenly changed the nature of the universe? Have I "caused" light to stop moving at a uniform speed. Have I changed the nature of light?


You have not changed the nature of light, nor yourself, nor nature of the universe. You have changed the relationship, the perspective-p, between yourself and any particular photon, which I suppose is a change in the universe (but not to its nature) on a localized scale. This change is real, but not within you nor the photon. It is within the universe, but not a change to its nature.

Think about it.

I do, carefully.

One Brow said...

Lightning strikes a cloud near me, causing a thunderclap sound. I hear the sound "now." A guy a mile away saw the lightning at (virtually) the same time I did. But he doesn't hear the sound until about 5 seconds later.

Question: Did the lightning strike and the thunder clap occur at two different times. One for the time I heard it, and another for the time he heard it?

A three year old might say "yes," no sound is created until you hear it. A "sophisticated" adult might say, yes, it's two different times, because "simultaneity is relative."

No reasonable scientist would ever say either, though. They would simple say that the lightning strike and the creation of the sound wave (the thunderclap) occurred at (virtually) the same time, regardless of subjective perception by different individuals.

What would you say?


First of all, and I don't know if this is deliberate, but you equated "the thunderclap" in paragraph 2 with "the creation of the sound wave" in paragraph 4. A thunderclap is not an instantaneous event, but a traveling sound wave.

However, I believe you meant the equivalent of "the creation of the sound wave" in paragraph 2. For any mainstream physicist, those you refers to SR proponents, the strike of the lightning that the creation of the sound wave are in a single point in spacetime. They have neither time-like nor space-like separation. Relativity of simultaneity only applies when there is a space-like separation. So yes, that happened at the same point in spacetime.

I said: "It's OBVIOUS that A and B agree, isn't it. They agree that he has $1. So they are in COMPLETE AGREEMENT! Q.E.D."

Often my responses to you are indirect, in the hope that I will stimulate you to actually think. That statement was a response to this one, of yours:

"The actual application of SR tells me they agree on everything. DTFM."

Do you see the connection? I doubt it, even if I make it explicit.


Sometimes I do forget to include the caveat. They agree on everything that can be directly compared; all the situations where you can verify you are seeing the same thing. They can disagree on calculations that can't be verified, like the current value on a clock many light-seconds away.

"Any mystical talk about "traveling through" spacetime is just self-deceptive hogwash....You get to the mall faster if you travel at a higher speed. Big whoop. Like, who knew?"

Relativist: "Can't you see the obvious, fool!? That means the distance is shorter! Space shrinks! Time, not clocks, slows down!"

I'm not sure exactly how to continue this pseudo-religious rant, because I don't happen to subscribe to this particular brand of religious dogma. Since you do, maybe you can't articulate it all for me, eh?


Time and distance don't change. Your path changes. When you change your path, you go through a different amount of path-time and a different amount of path-distance to get to the same point.

One Brow said...

I will note that you have also said the following things, at one point or another:

"Time doesn't slow down. Time is not a physical object that can move more quickly or more slowly."

We completely agree here.

"Distance doesn't expand/shrink. Space is not a physical object that can be larger or smaller."

We completely agree on this point also.

So why do we disagree on anything?

Because you don't really believe in, consistently apply, and adhere to those statements, that's why. You just pay lip service to them, then immediately commence to ignore and contradict them.

You do NOT pay any attention to the logical implications of your own claims.

As a result, you end up making contradictory claims, yet say they are entirely consistent.


Talking about the distance between two objects as something persistent is only meaningful when those two objects are not in motion with respect to one another, that is, in the same inertial frame. The distance between them is fixed, because the frame they are already in is the best frame for declaring their distance. The net distance does not change.

However, you can take different paths to get to cover that same distance. For example, say you are at the Krispy Kreme just south of the St. Clair Mall, and you decide to go up the St. Clair Cinema. If you go directly up 159, you'll go (at a guess) 2 miles. If instead, you head to the Walmart on 50 first, then take I-64 back to the theater, you'll travel about 6 miles. Your overall change in distance is about 1.8 miles in both cases, but you traveled a different length of path in one case versus the other. By the way, in all of our conversations, I don't recall you ever acknowledging this simple point; you seem to deliberate not respond to it instead. Why is that?

Talking about the time between two events is only meaningful when those events occur in very close spatial proximity to each other. The time between them is fixed, because the location they are in is the best point for determining how much time was gone through between the two events.

Just as it is consistent to accept the difference between net distance as a change in location (fixed) and the length of the path as a distance traveled, it is consistent to believe in the difference between a change in net time (fixed) and the length of the path as time taken.

Again, I am just explaining this, with no expectation that you accept it. However, if you want to claim there is a contradiction (which is different from simply saying you don't accept it) in differentiating between differentiating between net time vs. time traveled, you'll also wind up saying there is a contradiction between net distance vs. distance traveled. So, in my example above, you'll have to claim that the 6 miles driven is somehow a meaningless number. You have not undertaken this effort.

One Brow said...

In the wiki article on the twin paradox, this (among many other) claims is made:

"In their rest frame the distance between the Earth and the star system is εd = 0.6d = 2.4 light years (length contraction), for both the outward and return journeys."

Do you believe that, or do you believe, as you have claimed to, that: "Distance doesn't expand/shrink. Space is not a physical object that can be larger or smaller."

Notice that the parenthetical phrase "length contraction," as used by wiki, is inaccurate. They are NOT talking about length contraction. "Length is the distance between the beginning and end point of a particular object, like, say, a yardstick. The distance (space) between two separate objects is the distance (not length).

Wiki immediately transmutes the length contraction of the LT into a metaphysical "distance contraction" wherein space "shrinks."

Which view do you subscribe to? Does space shrink, as wiki claims, or not?


I think the wiki is very careless in its language. I do not think space actually, physically shrinks. I think you take a different path through spacetime. Most of the time, this carelessness is not relevant. However, for our particular discussion, this difference is highly relevant.

So, I believe in length contraction, not as a description of space itself, but as a somewhat metaphorical description of the difference in the path-length between two different paths in spacetime. Similarly, I believe in time dilation, but as a metaphorical description of the effects of taking a different path in spacetime, not as a description of time itself.

You have conceded that "measurements" can be inaccurate at times. At other times, you act as though whatever is measured is what *is.*

Which is it? Are you capable of making any kind of distinction between what "is measured" and what "is?"


Measurement always has error bars, and sometimes bias, but I don't think that is what you mean.

I used "see"/"observe" and "measure" differently. Measurement is the process of taking observations and correcting for any errors, and can also involve converting to a different perspective-p, if it has been determined that there is a preferred perspective-p. For example, if you decide the preferred perspective-p for deciding a person's height "standing right next to them", then a proper measurement involves making adjustments for the distance between you. If you decide the preferred perspective-p for height is "how much of my horizon is blocked by his figure", you don't need to adjust for the distance between you. It is not wrong for person A to say "person B obstructs five feet of my visual horizon"; it is a different context than saying "person B is 6 feet tall when you stand next to him".

Most of the time, we have cultural implications present for valid measurements. In this case, we decide that the real height is the height standing next to a person. Then, a measurement accounts for the difference between observer and subject.

So, before you can have a measurement, you have to have a context for what the measurement reflects. Once that context is established, measurements reflect reality to the degree allowed by bias and measurement error.

One Brow said...

Another way wiki could have put it would have been:

"Because it is NOT moving, the yardsticks on the spaceship are actually LONGER than those on earth. As a result of using longer yardsticks, they MEASURE the distance to be shorter. Of course the actual distance between the earth and the star remains unaltered, that distance is merely measured differently.

Because the earth IS moving, and the spaceship IS motionless, the earth mistakenly perceives the actual distance to be greater than it really is. But that is only because it is using distorted yardsticks."

Of course that would undermine the given premise, i.e., that the spaceship is actually travelling, and is NOT motionless. So which is it?


Let me ask you this way. A large spaceship leaves earth, and starts cruising at .5c. A thousand years later, the current population of the large ship has forgotten all about earth. Then, a much smaller ship takes off from the larger ship, and cruises at .5c compared to the larger ship in the direction of earth. Is the small ship really moving?

In your example, how can tell (that is, what experiment can you perform) to say the earth was not moving all along, and the spaceship simply decelerated and stopped moving while the earth kept moving?

Notice that wiki starts with this introductory phrase: ""In their rest frame"

"They" being the occupants of the travelling spaceship. But, if they are travelling, how can they be at "rest?"

Somebody is moving here, either the earth or the spaceship (or both). They cannot both be "at rest," even if they both think they are (or claim to be) "at rest."


In SR terms, "at rest" is shorthand for "within an inertial environment". You can be traveling and be, for almost all of the journey, inside an inertial environment. You can discuss what happens during the portion of the journey when you are in an inertial environment. So, in that sense, both can be in inertial environments, and both be "at rest".

So who is right? Who is moving, and who isn't? According to the standard resolution, the earth is the one who is "really" stationary. Therefore it is the travelling twin who will "really" age less.

At some point in the traveling twin's journey, he experiences acceleration, and so has no single rest frame that describes the entire journey.

This is just another way of saying that SR is in fact preposterous in it's self-contradictory claims (each is at rest) and that one must resort to an LR analysis wherein the earth is chosen as the preferred frame in order to arrive at the correct conclusion.

The earth is one possible rest frame. There are an infinite number of correct rest frames, but the earth's frame is easiest to make calculations from, so it is often preferred. The traveling twin is indeed not in a rest frame at all over the entire journey.

One Brow said...

Also notice that, from the viewpoint of the spaceship's "rest frame," as I said:

Because the earth IS moving, and the spaceship IS motionless, the earth mistakenly perceives the actual distance to be greater than it really is. But that is only because it is using distorted (shortened) yardsticks."

But notice: Now the earth has shorter yardsticks, not longer. But, by this hypothesis, it is also moving. But, if it is moving faster, then, according to the SR interpretation, the "distance" to the star would now be shorter, not longer.

This constant shifting back and forth between subjective viewpoints, treating each, in succession, as being "correct," just ends up in a never-ending series of inconsistencies. Not unexpected when you start out with a premise (both are at rest) that is logically inconsistent from jump street.


I agree you get different answers, in terms of which clock is slower or which ruler is shorter, from different frames. I disagree that there should be any expectation that the answers from different frames of events that are separated by space or time should be expected to be identical. I accept you have this preference.

What can we conclude from this? Well, they're both only 5 feet tall, I guess. Or they're both 6 feet tall, it you prefer. Or, neither is 5 feet all and neither is 6 feet tall, because such things can't possibly be known, maybe.

Or maybe the correct measurement depends upon what you are using the measurement for.

We can conclude that valid direct comparisons need to ber sone side-by-side.

One thing for sure though: each one is taller than the other.

Due to the (metaphorical) relativity of height, the taller person is viewpoint dependent, until you bring them side-by-side.

=====

Heh, and you claim to believe there is such a thing as "objective reality," eh? You are granting "reality" to the distorted perceptions of each observer here, that's all. You say "the taller person is viewpoint dependent." The viewpoint determines the facts, you say.


So you also say. You just have a very specific viewpoint in mind when you say it.

If "valid direct comparisons need to ber sone side-by-side," as you say, then all science is impossible. No two measurements made at different times, in different parts of the world, could ever be compared. If one guy measures the distance from Chicago to New Orleans to be 700 miles, no other guy can ever make a measurement which would constitute a "valid direct comparison." It wouldn't matter if he measured 700 miles, 699, 701, or whatever. You couldn't possibly compare the two, they were not made "side by side."

In this case, given that tectonic plates shift even within continents, the distance itself may change.

More to the point, consistency of measurements throughout time is an assumption of science. It can not be demonstrated. So, we assume the guy measuring later should have a result directly comparable to the guy measuring earlier, but it can't be demonstrated.

That said, two people standing next to each can see the same number on the same clock, and the same number on a ruler, at the same time.

One Brow said...

You have given the WRONG answer to this question, sorry. Hogg gives you the right answer. You must correct for differences which are due to inessential matters emanating from subjective perspective, and THEN compare. You don't abandon any and every attempt to compare measurements as being "invalid" if not taken "side by side."

If you want to compare, absolutely you have to transform a measurement into the preferred context. The difference between SR and LR is the allowable number of contexts.

You have, of course, advanced the standard evasion of the positivistic relativist and the creed he adheres to (while constantly violating in practice). It's' "meaningless."

You don't understand "creed" of SR well enough to understand what principles violate it.

I said: hey each get out their yardstick and "measure" how tall the other guy is at that distance. A determines that B is now only 5 feet tall. Guess what!?

B ALSO measures A to be only 5 feet tall!!!

All this while A and B both perceive themselves to be 6 feet tall!!

They get the exact same difference!"

I was being facetious here, but you say:

"Exactly."

====

Exactly, what? I suspect you mean this proves they get the "same answer." But, as I have said. B says A is shorter than him. A says B is shorter than him.

This is NOT the same answer being given by both A and B. The "same difference" is not the "same answer."


Exactly, meaning there is no reason for each, from their own perspective-p, to assume his measurements will be identical to those made in a different perspective-p.

"There is no SR calculation where the sun clock runs slower than the earth. If you really believe otherwise, prove it. Run the actual numbers. Show me."

Heh, how can you even think that this is an open question and still claim to understand the underlying concepts of SR? This just shows how completely you confuse LR with SR.


Because, from any inertial frame, over the course of a year, the motion of the sun will be less than the motion of the earth.

i am on earth. I am in my own inertial "rest frame."

If we are talking about motion on the scale of the sun and earth, than the earth is in no ways in an inertial rest frame. Again, there are observations you can run to verify this, such as stellar parallax and planetary retrograde motion.

I am going to employ SR to make time dilation calculations. With respect to me, the sun is moving (per SR, and the very definition of a "frame of reference, I am NOT moving, the sun is). I am adopting, as I must in SR, a "geocentric" view of the universe.

Since, from my frame, the sun is moving, then, per the LT, IT'S clocks have slowed down. Why? Because it is the one moving.

You really don't understand that this is what SR dictates?


I really understand this is not what SR dictates. I really understand that, by the first principle of SR, you have to have an inertial environment, or at least an inertial frame, to claim you are not moving. I know that, from that perspective, the Sun is much closer to exhibiting inertial movement than the earth.

One Brow said...

Several posts back I ended with this statement: "Take Fowler's example. Jack says his clock reads 10 and Jill's 8. Following SR, IF she were the one at rest, then her clock would read 10 and his 8. But they would NEVER agree that their respective clocks give the same readings for both of them. NEVER. It's not allowed in SR. In LR they would always agree, but that's LR, not SR."

Actually, it's both, for any direct comparisons.

Knowing you, you will say they did agree. They didn't. Sure they did (necessarily) agree about what they both saw at the same time and the same place. It should have ended there. Jill should have realized that SHE was the one moving, whether she previously knew it or not.

But Jill does NOT do that. In effect, she DENIES that Jack's clock reads 10. She says it "really" reads 6.4 ls, not 10.


Incorrect. When Jill passes clock1, she sees a -6 on Jacks clock, and can calculate that Jack's clock currently reads 3.6 seconds based on the speed of Jack/clock1 and the distance between them. When she passes Jack's clock 8 seconds later, it has ticked off another 6.4 seconds, and currently reads 10.

Of course this makes no sense, either, but don't blame me, blame Fowler.

I blame you for not thinking through the situation carefully enough, and Fowler for adopting a false sense of absolutism in his explanation.

The two clocks are conceded to be synchronized in Jack's frame. One can't read 6.4 and the other 10. But of course Fowler's acts like he has made sense out of, and has "explained," why two clocks, when synchronized, can have different readings.

They don't have different readings. Jack's clock reads 10, in both Jack's calculations and in Jill's.

This is just another "If you're going 89 mph, how long does it take you to go 80 miles?" kind of question:

Q: If two clocks are synchronized, and one reads 10, what does the other read?

A: (from Jill, the buffoon) 6.4!!

Really, Jill? Why do you say that? SR makes me say it, that's why.


What makes Jill predict a clock to say 10, see a 10 on the clock, and call it 6.4? aintnuthin gonna do that.

That last post was just an elaboration on the preceding one which it referred to. I am trying to preempt what I think your misconception will be.

The main point is the same: In SR, two observers who are moving relative to each other will NEVER (per the dictates of SR, as a "theory") agree on what the other's clock reads.

You think otherwise, but only because you do not understand even the most rudimentary principles of the theory.


I think otherwise, because the LT force it to be so. You don't think otherwise, because yo udon't understand the theory and won't do even the most basic math to attempt to understand it.

No math required.

Given the reults no math have produced for your understanding, math is certainly highly recommended.

Jack and Jill will agree on one thing: Her clock reads 8. But this is only because she is the one moving, which she denies.

They also agree his clock reads 10.

One Brow said...

But this doesn't mean they "completely agree on everything" as you assert. Jack will say (know) that the first clock also reads 10, because the two are synchronized in his frame. Jill will claim it reads 6.4 (which is the equivalent of saying the one she is looking at, which reads 10, "really" reads 6.4).

Incorrect. Jill does not, from her own frame, see Jack and clock1 as being synchronized. They don't read the same numbers to her.

Why? Because she is a denialist who CANNOT and WILL NOT be deterred from her assertions by either facts or logic. In SR she MUST deny that she is moving (which would mean that, if her clock reads 8, then Jack's clock *HAS* to read 6.4, irrespective of what she sees).

If Jill insists she is not moving, using SR will lead her to say Jack's clock reads 10 when she passes it.

I already know that it is extremely unlikely that you will be able to comprehend this simple demonstration.

I comprehend the demonstration you are attempting to make. However, your demonstration suffers from a fatal error. Your demonstration overlooks that, when Jill passes clock1 and sees the -6 on Jack's clock, that this number means, to Jill, that Jack's clock currently reads 3.4, not 0.

Some people seem to think they are saying something very profound if they say "simultaneity is relative."

They are not. They also like to act like they know exactly what they are talking about when they use that phrase, when they don't. I explained, in very simple terms, the difference between relative and absolute simultaneity.


Yet, you completely failed to apply it appropriately. Relativity of simultaneity refers to events with a space-like separation, such of clock1 and Jack. It means that observers in different inertial frame, as Jack and Jill are, measure different events to be simultaneous. Jack measures his clock to be 0 as being simultaneous with clock1 being 0. Jill measured Jack's clock striking 3.4 as being simultaneous with clock1 striking 0. You keep spouting off as if, to Jill, Jack's clock reads 0 at the same time as clock1, when this *can not* be true if Jill sees a -6 on Jack's clock as she passes clock1.

Your only comment, as I recall, was to say that you didn't agree with my interpretation. Of course you didn't say what *your* interpretation was, and I don't think you would be able to give an alternative interpretation. If you did, it would be wrong.

I'm sure you think that.

One Brow said...

I am not talking about "my" interpretation to begin with. I am talking about Einstein's interpretation, as explained in his own words, using his own illustrative examples.

Al's example is a lot like Fowler's, and just as fallacious.

1. Al posit that one guy (on a train) is moving and another, by the tracks, is stationary relative to the earth's surface. OK, now what

2. He uses his knowledge of their motion to explain why the light from two different lightning strikes reach the eyes of the two observer's at different times.

OK, fair enough, an quite easy to understand. This is just a case where you should correct for the perspectival difference created by then motion, and then compare.

The guy on the train should, when calculating whether the two strikes were simultaneous, need only (and should) calculate the consequences of his motion and factor that out. If he does that HE, like the guy by the tracks, will say that the two strikes WERE simultaneous.

End of problem. Very simple actually. Al has told us who is moving. After factoring this in they AGREE that the strikes were simultaneous.

One problem. To get his fancy-ass notion of "relative simultaneity" he has to prohibit this simple correction. How does he do that. Easy, even though HE knows the guy on the train is moving, he prohibits the guy on the train from acknowleding that. He must FALSELY insist that he is stationary while everything attached to the earth's surface whizzes past him. Al KNOWS this is false, but he nonetheless forces the train passenger to adopt this mistaken claim.

As I have said (as as Morin said), Al must insert a fool into the situation who doesn't know that he boarded a train and, for that reason, is now moving relative to the earth. Only by creating a chump, who is prohibited from knowing what Al knows, can he claim that, "simultaneity is relative." It aint. The fact that the train passenger, being a fool, "sees" the two as non-simultaneous is irrelevant. He is mistaken in what he thinks explains what he sees."


Unless it was the earth moving all along, and the train that decelerated to a complete stop. Then, it turns out that the fool on the earth is wrong, and the guy on train is right, about whether the events are simultaneous. What experiments can they do to show that the earth was not originally moving, and that the train accelerated rather then decelerated?

You might just as well say that if I see a supernova explosion, which occurred long ago, and far away, now then it happens NOW, for me. And, if it happened now, for me, then, BY GOD, it happened NOW, not before. I am too ignorant to understand that it COULDN'T have happened now, if I see it now.

We have measured that light has speed. There are no measurements that reveal the "true" rest frame.

Only an idiot would think that what he "sees" is, by necessity, what IS TRUE. Or, just replace "idiot" with "solipsist." Same thing, in my book

What's true can be well beyond what we can measure.

Let's say the star was 10 million light years away when it went supernova. A guy who was there at the time would say it happened 10 million years ago. Now suppose I'm NOT an idiot who thinks everything is all about me and that it could have only happened at the very instant I saw it. If I say that, as a solipist, then I am saying, quite profoundly, no doubt, that "simultaneity is relative").

Wow, that a stunning ignorant depiction of the relativity of simultaneity. I assume this is hyperbole.

So what do I say if I'm not an idiot? I say to the guy who was there: "I agree with you. This event occurred 10 million years ago."

What's that called, if we agree? It's called "absolute simultaneity."


It's called SR, because in SR, you always agree.

One Brow said...

Anyone who claims that Einstein "proved" that "simultaneity is relative" has about as much sense of what "proof," and "science," is as someone who claims that whatever conclusions he wants to draw from a compilation of plus/minus data in a basketball game are "scientific."

As physicists say, proof is for mathematics and alcohol.

So, you ask, what practical difference does it make if you use SR (which posits that simultaneity is relative) or LR (which posits absolute simultaneity))?

I just gave you one "practical difference." If simultaneity is relative, then I say any supernova I see, anytime, "happened" at the exact moment I saw it.


This is NOT what the relativity of simultaneity claims.

This is a rather naive view of "science," eh?

It a completely wrong depiction of SR.

It in fact PROHIBITS science. As Al noted, physics MUST presuppose the existence of an "objective reality." But the view taken here DENIES that there is any objective reality. The only "reality" that exists in this view is strictly subjective. It is, as Al also noted, solipsism, which precludes science (because physics must presume the existence of an objective reality, in addition to a "subjective reality").

In SR, it's not that there is no objective reality (in terms of an absolute rest frame), but that you can't determine what it is by experimentation.

All said and done, Eric, it's not very complicated. If you could ever grasp the following two points, you might be on the way to understanding SR, LR, and the difference between the two:

1. What you "measure" something to be (time and distance for example) is not necessarily "true." If your measuring instruments are distorted (by heat, cold, motion, whatever) then you will not get the "true" (as established by the accepted etalons) dimensions of what you're trying to accurately measure (time or distance).

2. Subjective reality is different than objective reality. The two will not necessarily lead you to the same conclusions. Subject reality is NOT the equivalent of, and identical to, objective reality.


All said and done, aintnuthin, if you could ever grasp that I do, in fact, accept that measuring with distorted instruments can lead to distorted results, and that I, in fact, distinguish between an objective reality, a subjective reality, and even in-between realities like the reality of perspective-p, and that *none* of this is in conflict with SR, then you might be on the way to a genuine understanding of SR.

aintnuthin said...

"You are now making the opposite error I did. The underlying assumptions are important, but they can only be judged by their ability to make successful predictions, and to make those predictions, you have to DTFM.

I learned from my error."

No, Eric, you are simply repeating the error. Applying the Lorentz transformations is a simple mechanical operation, which, in itself, adds absolutely NOTHING to the conceptual underpinnings. HOW did Lorentz derive his equations? What suppositions were they based on? These are the concepts, not the formula derived therefrom. DTFM math adds NOTHING.

====

"By contrast, you find some obscure people advocating for some minority viewpoint, often based on completely misunderstanding the topic and hand, and then bray loudly about how 99% of scientists are misinformed and ignorant."

Quite typical of you to impute your misconceptions to 99% of scientists. I can't give you the percentages, but I think that many, if not most, scientists are aware that LR and SR give the same predictions (about certain, limited things). That is not a minority viewpoint and I assume that most scientists are well aware of it. DTFM, eh? The math is identical in SR and LR. The differences are CONCEPTUAL, not mathematical. But like many, you seem to think that math is physics.

Also, like many everyday physicists, you seem to have little background, if any, in the PHILOSOPHY of science. They are trained to make mathematical calculations but often have been given little training in the fundamental theoretical aspects of their topic.

Every experiment that "confirms" SR ALSO confirms LR. Yet I have seen big-name physicists actually claim that SR is not a theory, but rather a "fact." Apparently he never even heard of the concept of "underdetermination." Such people obviously do not understand (or, rather, don't even think about) the conceptual difference in applying theoretical SR and applying theoretical LR. They think that if the LT generate accurate predictions, then SR has been proven. The LT came from LR, not SR.

aintnuthin said...

I can remember you claiming that if the AVERAGE speed of an object over a certain distance was 100 mph, then that proved that it was travelling at 100 mph at every instant over the duration of it journey. This is a very fundamental conceptual mistake. What you say is, of course, "possible," but it is by no means a necessary logical implication.

Do the math all day long, and, it will show you that if an object is travelling at 100 mph at every instant, then its AVERAGE speed will ALSO be 100 mph. But, so what? It is a basic misconception to draw the conclusions you did from this tautology. If all elephants are purple, and if this animal is an elephant, then, by logical necessity, this animal is purple.

That's math (logic, same thing, really) for you. Math will "prove" your assumptions. It has to. You have built the answer in when you formulate the question. But there is a big difference between a logical argument being "valid" and being "sound." You often act as if "validity" proves "soundness."

Needless to say, this indicates a flawed understanding of the nature of logic.

One Brow said...

No, Eric, you are simply repeating the error. Applying the Lorentz transformations is a simple mechanical operation, which, in itself, adds absolutely NOTHING to the conceptual underpinnings. HOW did Lorentz derive his equations? What suppositions were they based on? These are the concepts, not the formula derived therefrom. DTFM math adds NOTHING.

Among other things, DTFM allows you to add implications of the theory, like the invariance of observations.

You still haven't listed/discussed the complete underpinnings of LR. By contrast, I have listed and discussed the complete underpinnings of SR (there are only the two, besides the universal underpinnings of scientific theories in general). So, you have not even lived up to your own meager standards. One is your complete list of the underpinnings of SR?

Quite typical of you to impute your misconceptions to 99% of scientists. I can't give you the percentages, but I think that many, if not most, scientists are aware that LR and SR give the same predictions (about certain, limited things). That is not a minority viewpoint and I assume that most scientists are well aware of it.

Just one round ago, you were discussing how SR meant two obervers in different inertial frames (such as Jack and Jill) never agreed on anything. Have you changed your stance? If not, what are the sorts of things you think Jack and Jill agree on in LR and in SR?

DTFM, eh? The math is identical in SR and LR. The differences are CONCEPTUAL, not mathematical. But like many, you seem to think that math is physics.

I've been saying all along that LR is just SR plus a preferred point of view. That implies the math would indeed be the same.

Also, like many everyday physicists, you seem to have little background, if any, in the PHILOSOPHY of science. They are trained to make mathematical calculations but often have been given little training in the fundamental theoretical aspects of their topic.

Every experiment that "confirms" SR ALSO confirms LR.


Agreed.

One Brow said...

Yet I have seen big-name physicists actually claim that SR is not a theory, but rather a "fact."

In the same way that evolution is a theory and a fact, so is SR. The experience of length contraction and time dilation is an observed fact; the theory of SR brings together these observations with an explanation for them, creating a theory.

Apparently he never even heard of the concept of "underdetermination." Such people obviously do not understand (or, rather, don't even think about) the conceptual difference in applying theoretical SR and applying theoretical LR. They think that if the LT generate accurate predictions, then SR has been proven. The LT came from LR, not SR.

1) The LT only require the two principles of SR to be generated. So, the LT come out of SR just as much as they come out of LR.

2) LR and SR are not competing theories. In every possible experiement, the verifiable predictions of LR and SR are the same.

I can remember you claiming that if the AVERAGE speed of an object over a certain distance was 100 mph, then that proved that it was travelling at 100 mph at every instant over the duration of it journey. This is a very fundamental conceptual mistake. What you say is, of course, "possible," but it is by no means a necessary logical implication.

If you replace "every instant" with "some instant", and assume that changes in speed happen gradually instead of in bursts, this is true. It's certainly not true as stated, and it doesn't sound to me like anything I would have said.

Do the math all day long, and, it will show you that if an object is travelling at 100 mph at every instant, then its AVERAGE speed will ALSO be 100 mph. But, so what? It is a basic misconception to draw the conclusions you did from this tautology. If all elephants are purple, and if this animal is an elephant, then, by logical necessity, this animal is purple.

Agreed.

That's math (logic, same thing, really) for you. Math will "prove" your assumptions. It has to. You have built the answer in when you formulate the question. But there is a big difference between a logical argument being "valid" and being "sound." You often act as if "validity" proves "soundness."

I believe that you read me this way.

Needless to say, this indicates a flawed understanding of the nature of logic.

I agree that the description you provide indicates a flawed understanding of logic.

aintnuthin said...

"I showed that, regardless of whether A, B, or C is really at rest, the comparable numbers always come out the same; so you can't, by comparing clocks, say who is really moving."

All I do is repeat myself, but I'll try this once again:

1. LR does NOT have multiple "motionless" frames, only SR does. You are not even talking about SR.

2. What "comparable numbers?" What you're talking about it irrelevant, and doesn't prove what you seem to think it does. If you can't see this immediately and intuitively, then it probably can't ever be explained to you in a way that you understand it (I've already done that many times over, directly and indirectly).

You simply say that *SOME* (but not all) numbers are the same in different frames. So what? It does not prove, and in fact disproves, your claim that all frames give identical results.

3. If you accept the validity and accuracy of the LT, you can tell who is "really" moving by simply observing which clocks record more, and which record less, time elapsed.

3 clocks involved in the HK experiment. EACH clock showed a different amount of time elapsed. For simplicity Let's just say that one read 100 hourse elapsed, one 99 hours, and one showed 101 hours elapsed.

Using LR, all "observers" will agree on the predictions: Clock A will show 101, Clock B will show 100, and clock C will show 99. That will not be how ONE of the three clocks "sees" it, they will all "see" it exactly that same way. It won't be that *some* frames of reference see *some* readings to be the same, as your example appears to stress. You might just as well say that since both dogs and elephants have four legs, they are identical creatures.

It is unfathomable how you could believe that, as you have previously claimed, in SR all inertial frames of reference give you the exact same answers for each of three clocks. It just shows you know NOTHING about SR.

aintnuthin said...

Edit: Meant to say: "You are not even talking about LR" (not SR).

aintnuthin said...

"You can use the ECI in the HK experiments to get the right answer. You can use the SCI (the math is harder, but it can be done). You can use the CMB. These are three different frames, all effectively inertial for the purposes of HK, and they all give the exact same answers for the times shown on the planes and on the earth-bound clock. Not just the same differences, the EXACT SAME NUMBERS."

Simply wrong. Can you treat, for example, the clock stationed at the naval observatory, as your "preferred frame" and get the "same answer?"

Hell no. If you do that, then both of the airborne clocks would slow down in comparison (by equal amounts). That's NOT what happened. One slowed down, one sped up.

aintnuthin said...

Let's look at the Michelson-Morley experiment in simplistic terms for a minute. What was the "problem?" What was the suggested "solution" (explanation)?

We start with the assumption that the earth "really is" moving in a orbit about the sun.

If that's true, then we would expect light to take longer to travel between 2 co-moving points on the earth if it's travelling "with" the motion of the earth as opposed to "against" the motion of the earth. Why? Because it has to travel a different distance, depending on which way it is travelling. This was the basic assumption of Al's "simultaneity is relative" example involving the guy on the train.

But our experiment (M-M) does not produce this result. Now what?

1. One possible answer: The earth is NOT moving. It is a geocentric universe after all. For some reason, this was not deemed to be an acceptable answer. For some damn reason, contrary to the premises of SR, scientists believed that we could, and in fact had satisfactorily, determine(d) that the earth is "really moving."

2. OK, then what is the explanation? Another possibility: IF clocks slowed down by a certain degree, and if our measuring instruments shortened by a certain degree, THEN we would MEASURE the speed to be the same, even though it wasn't "really" the same.

Hence the LT.

The whole premise is that we are getting identical *measurements* of a phenomenon that is not "in fact" the same. This proposed solution presupposes that the underlying "reality" is different than what we empirically *measure" it to be.

Far-fetched, you say? Ad hoc, you think? Maybe. But that's the premise. Maybe the "real" solution is that the whole universe is, in fact, geocentric.

aintnuthin said...

To me, SR is similar to the cosmological argument for the existence of God.

This argument starts by getting you to conceded that every event has a "cause" that precedes it in time.

But then it says this assumption logically leads to an unsatisfactory "infinite regression."

Therefore, it says, there must be a non-contingent "Prime Mover" or "First Cause" (which we can call God).

But notice that the solution requires that you reject the very assumption that it is based on, i.e., that every event has a pre-existing cause. But I reject that premise, then there is no force to the argument to begin with. You want me to say that "x" is true, and then use that assumption to prove that "x" is not true. See the problem?

SR does the same thing. It uses the LT. The LT presuppose that "real" motion does exist, and that "inaccurate" measurements from instrumental distortion result by a factor set forth in the LT. It supposes that there is an "objective reality" which is DIFFERENT from what is suggested by our empirical measurements.

Based on your accepting the premise that there is "real" motion, and that there is an "objective reality" that is at variance from what we actually measure, it then tries to tell you that there is no such thing as objective reality or "real" motion. Which one is it?

Don't try to persuade me that insisting that I affirm a condition that you deny is true.

aintnuthin said...


As I said before, if you want to insist that the speed of light REALLY IS constant in all frames of reference, THEN you would just throw the LT out the window. There would be no need for the LT to "explain" why you measure the speed of light to be the same. In that case you would "measure" it to be the same, simply because it IS the same. It could only both BE the same, and be MEASURED to be the same IF they are never any distortion to measuring instruments cause by relative motion.

But of course this simplistic assertion leads to contradictions elsewhere. In effect it presupposes that nothing is ever moving, and, at the same time, that everything is moving. SR has to build in inherent conflict, where every observer denies the "reality" of the others perspective. Every observer must say only the "other" guy is moving and that only the "other guy's" clock slows down, etc.

The logic inconsistency is obvious. Both cannot be right in their respective claims.

aintnuthin said...


"The logic inconsistency is obvious. Both cannot be right in their respective claims."

You try to deny that there is any inconsistencies, notwithstanding the fact that virtually every scientist (along with every other rational person) denies that "each clock can actually run slower than the other."

How can you do this? Only by taking a position of utter, unmitigated solipsism. There is no objective reality whatsoever. There is no "truth." The only "truth" is a subject believes is true. If two subjects believe different things, then that means there are are two absolute truths, one for each subject.

aintnuthin said...

"The only "truth" is a subject believes is true. If two subjects believe different things, then that means there are are two absolute truths, one for each subject."

This position is prima facie absurd, but that's not the end of it. SR exacerbates this inherent absurdity. The basic premise that what you "believe" to be true is, ipso facto, actually true. Now SR has to take an EXTRA step to complete it's "theory." It has to FORCE observers to "believe" things that are inherently incredible in order to create the necessary conflict. Once that forced acceptance of obvious falsity is imposed, NOW, and only now, can the theory remain consistent.

The basic premises come down to this:

1. Whatever any subject asserts to be true is, by definition, actually true.
2. What they "say" is true is in fact true, even if they assert that something is true, when they know it is not. It is only their assertions, no matter how insincere, that can be counted in ascertaining what is "true."

Sorry, I aint buyin.

aintnuthin said...


Parmenides, already discussed, who claimed that all motion is an illusion is one Greek whose views are effectively adopted by relativists. But he's not the only one. Another is the sophist Protagoras.

Protagoras said something like: Man is the measure of all things; of those that exist, that they exist, of those that do not exist, that they do not exist."

A very curious mixture of absolute "Being" (which denies "becoming") with absolute relativism, where there is no objective truth whatsoever. "Man is the measure of all things." Of course the two are incompatible, but that doesn't stop people from failing to see the incompatibilty, eh?

aintnuthin said...

Did you watch the "how long does it take to go 80 miles" video where the gal was talking about how many times a tire rotates in a minute (which is relevant to how odometers operate, of course) and how much air was in the tires (also relevant to the accuracy of odometers)?

She was trying to do elaborate calculations, based upon estimates, to arrive at the simple and obvious answer of "one hour." She could not distinguish the mathematical details of an entirely different question from the simple, non-mathematical, question being asked.

Of course she also told her interrogator that he himself did not know the (self-evident) answer to the question he was posing. DTFM, eh?

aintnuthin said...

I didn't comment on it at the time, but it appeared that you thought you were proving the "ignorance" of a Harvard physics major, by implying that he did not understand the concept of velocity.

I have shown you, time and time again, that a "third postulate" (the clock hypothesis) holds that ONLY speed (not velocity) affects time dilation. Although the concept of an "inertial frame" is relevant to SOME aspects of SR, it is irrelevant to the question of time dilation. In that matter, acceleration is totally irrelevant.

You also called Louis Essen and Albert Einstein "idiots" for not agreeing with your misconception about this issue. Only YOU (well, and 99% of non-crank physicists, of course) know the real truth about this matter, eh?

LR makes accurate predictions about the time dilation of ALL objects in all frames, not just inertial ones. It applies equally to accelerating frames. You claim that SR ONLY applies in inertial frames, and then make the absurd claim that LR is just a "special case" of SR. At best, the exact opposite is the case, i.e., SR is just a "special case" of LR. If you really understood the difference between the two, this would be obvious to you.

aintnuthin said...

To refresh your memory, here is it for about the 6th or 7th time:



"Does a clock's acceleration affect its timing rate?

It's often said that special relativity is based on two postulates: that all inertial frames are of equal validity, and that light travels at the same speed in all inertial frames. But in real world scenarios, objects almost never travel at constant velocity, and so we might never find an inertial frame in which such an object is at rest. To allow us to make predictions about how accelerating objects behave, we need to introduce a third postulate.

This is often called the "clock postulate"....So the clock postulate says that the rate of an accelerated clock doesn't depend on its acceleration....

... it's a statement about the physical world. But we don't know if it's true; it's just a postulate...Although the clock postulate is just that, a postulate, it has been verified experimentally up to extraordinarily high accelerations, as much as 1018 g in fact."

http://math.ucr.edu/home/baez/physics/Relativity/SR/clock.html

aintnuthin said...

As I have also repeatedly reported to you, physicists concede that the aspect of SR that is abandoned with respect to accelerating objects is the postulate pertaining to the constancy of the speed of light (as measured). This also requires the abandonment of the absurd claim that all clock retardation is "reciprocal." The "reciprocity" claim in a necessary consequence of accepting the absurd notion of "relative simultaneity."

Or, conversing, acceptance of the absurd, solipsistic notion of relative simultaneity, requires that clock retardation must be declared to be reciprocal, even though that is logically impossible as a matter of objective reality, i.e. as a matter of PHYSICS (as distinguished from math). In SR every (non-accelerating) observer in relative motion with respect to another observer MUST make conflicting assertions about the other. Unless, that is, one of them is accelerating, and then, suddenly, there is no reciprocity. Time dilation suddenly "becomes" directional.

All this is just another way of saying that SR adopts (and in fact "turns into") LR in those cases. I know that you don't understand a word of this, so I ask myself why I bother.

aintnuthin said...

"As I have also repeatedly reported to you, physicists concede that the aspect of SR that is abandoned with respect to accelerating objects is the postulate pertaining to the constancy of the speed of light (as measured)."

Notice that this postulate is entirely independent of the LT. The LT have been confirmed, this postulate has not been confirmed. The postulated "reciprocity" of time dilation has NOT be been confirmed. In fact it has been effectively disproven by observations pertaining to the GPS and H-K type experiments.

Again, it is no surprise that "reciprocity" (the postulated relativity of simultaneity) has not been "confirmed," inasmuch as it is logically impossible as a matter of physics.


aintnuthin said...

"Just one round ago, you were discussing how SR meant two obervers in different inertial frames (such as Jack and Jill) never agreed on anything."

No, I never said any such thing. They no doubt agree on lots of things, that Jack's name in Jack and that Jill's name is Jill, for example.

What they are prohibited, by fiat, from agreeing on is the question of which one is moving. With SR, Jack must say it is Jill, and Jill must say it is Jack.

This further implies, for example, that they would NEVER agree on whose clock was slower. No matter what evidence to the contrary might present itself, they CANNOT agree on that. Not if they "believe" in SR, anyway. If they do, then LR is being applied, not SR. To agree implies a shared perception that some frame of reference is preferred.

aintnuthin said...

That does NOT mean that there is no preferred frame for either of them. Quite the contrary. There is a preferred frame for each of them that each adamantly insists is the only correct one. The preferred frame is the one that they are in, that's all.

aintnuthin said...

"The preferred frame is the one that they are in, that's all."

You can't really call that "geocentric." You'd have to call it egocentric. Such is the nature of abject solipsism and "objective relativism." You also don't seem to comprehend that "objective relativism" is a contradiction in terms. Relativism is inherently SUBJECTIVE, never "objective."

aintnuthin said...

"Using LR, all "observers" will agree on the predictions: Clock A will show 101, Clock B will show 100, and clock C will show 99."

Perhaps I was not explicit enough in stating what I took to be obvious (and which I have already pointed out). When I say "using LR," I do NOT mean that you can just pick, willy-nilly, any old frame of reference, posit it to be the preferred one, and still generate predictions which comport with the observed facts. You CAN'T do that.

Every frame of reference you choose will generate different predictions. So the question becomes "Which (unique) frame of reference needs to be used in order to generate predictions which match the observed facts?"

In the case of the H-K experiment, which took place in a location where the earth itself is the dominant gravitional center, the answer turns out to be the ECI. It is deemed to be the preferred frame. Not because of some arbitrary, whimsical free choice, but because it is the frame which gives the right answers.

You get the correct answers (predictions) when you determine how the various clocks are (absolutely) moving with respect to (relative to) the ECI, NOT with respect to how the clocks are moving relative to each other, as SR would require.

aintnuthin said...


When Newton determined the orbits of the planets in the solar system he chose, as his "preferred frame," the barycenter of the solar system. He then chose, as a "close approximation" of a stationary background, the "fixed stars." Using these components, his "celestrial mechanics" turned out to be exceedingly accurate.

Had he used the earth, or Mars, or the Sun, or any other solar object as his "preferred frame" he would have gotten completely different "answers."

Think about it.

One Brow said...

General points:
1. Time and again, I say that comparable results are invariant in different inertial frames. Time and again (admittedly not every time) in your response, you ignore the word "inertial". I don't know if this is deliberate or not. Either way, I am not adding the word "inertial" to be typing more letters. I add it because it matters. You claim to be arguing from principles rather than math. The first principle of SR specifically restricts SR to discussing events from the viewpoints of inertial frames. Every time you talk about the frame of, for example, one of the clocks in the HK experiment, you are discussing a non-inertial frame. SR makes no predictions about the overall measurements made from non-inertial frames. As you point out, the second principal of SR is violated in non-inertial frames. If you are really arguing from the principles, that would include arguing about SR from the actual principles of SR, would it not?
2. You have again brought up the lack of effect of acceleration on clock speeds. For the purposes of this discussion of SR/LR, this is not relevant. You know that, I know that. I do not bring up acceleration because it affects clocks (for the purposes of this discussion). I bring up acceleration when you try to use SR (instead of GR) from the point of view of an object under acceleration. These are not the same thing.
3. You regularly claim I don't understand some point, but the only evidence seems to be my disagreement on an implication. Do you honestly believe that anyone who properly understands the topic must agree with you? Do you feel you possess that level of mastery? If not, can you be more specific regarding why you think I don't understand a particular topic, when you mention it?

"I showed that, regardless of whether A, B, or C is really at rest, the comparable numbers always come out the same; so you can't, by comparing clocks, say who is really moving."

All I do is repeat myself, but I'll try this once again:

1. LR does NOT have multiple "motionless" frames, only SR does. You are not even talking about LR.


That's why I expanded the scenario to having A1, B2, and C3 all be in the same "motionless" frame. There is only one "motionless" frame discussed in the post. It does indeed discuss LR.

2. What "comparable numbers?"

Clocks in the same place at the same time. (Also rulers in the same inertial frame, but that's not relevant to this example).

What you're talking about it irrelevant, and doesn't prove what you seem to think it does. If you can't see this immediately and intuitively, then it probably can't ever be explained to you in a way that you understand it (I've already done that many times over, directly and indirectly).

You claimed that different inertial reference frames predict different numbers on the clocks. I demonstrated this is false.

What do you think I was aiming to demonstrate ("prove", in your paragraph)?

You simply say that *SOME* (but not all) numbers are the same in different frames. So what? It does not prove, and in fact disproves, your claim that all frames give identical results.

They give identical results for all numbers that can be directly compared.

One Brow said...

3. If you accept the validity and accuracy of the LT, you can tell who is "really" moving by simply observing which clocks record more, and which record less, time elapsed.

If you accept the validity and accuracy of the LT, you can never tell whose *inertial* frame is "really" moving by simply observing which clocks record more, and which record less, time elapsed, because the LT have a spacetime invariance built into them.

These two following claims are incompatible. You can only consistently put one of them forth:
1) Choosing different inertial frames gives different predictions on what two clocks will read when you compare the clocks while they are next to each other.
2) The LT are valid and accurate.

The truth of the second negates the truth of the first.

3 clocks involved in the HK experiment. EACH clock showed a different amount of time elapsed. For simplicity Let's just say that one read 100 hourse elapsed, one 99 hours, and one showed 101 hours elapsed.

Using LR, all "observers" will agree on the predictions: Clock A will show 101, Clock B will show 100, and clock C will show 99.


Using SR, all observers will agree on those predictions as long as they base their observations in some inertial frame, regardless of choice of inertial frame.

That will not be how ONE of the three clocks "sees" it, they will all "see" it exactly that same way.

None of the clocks are in an inertial frame, so each of the 'clocks' (that is, an observer with that clock) will need to pick an inertial frame that observer is not in to work from. Even if they choose three different inertial frames, they will all have the same predictions. Not the same differences, those exact same 101, 100, 99. It's baked into the LT.

It won't be that *some* frames of reference see *some* readings to be the same, as your example appears to stress.

In every inertial frame, every direct comparison reads exactly the same. The numbers that are different can't be compared for accuracy.

It is unfathomable how you could believe that, as you have previously claimed, in SR all inertial frames of reference give you the exact same answers for each of three clocks. It just shows you know NOTHING about SR.

That you believe it could be any other way shows that you don't understand much about the LT. Any theory that uses the LT to transform from one reference frame to another will have these invariant spacetime points.

"You can use the ECI in the HK experiments to get the right answer. You can use the SCI (the math is harder, but it can be done). You can use the CMB. These are three different frames, all effectively inertial for the purposes of HK, and they all give the exact same answers for the times shown on the planes and on the earth-bound clock. Not just the same differences, the EXACT SAME NUMBERS."

Simply wrong. Can you treat, for example, the clock stationed at the naval observatory, as your "preferred frame" and get the "same answer?"


You can't accurately use SR from the frame of that naval observatory clock at all. The clock in the naval observatory in not in an inertial frame. This lack of an inertial frame can be demonstrated by experiment.

One Brow said...

Hell no. If you do that, then both of the airborne clocks would slow down in comparison (by equal amounts). That's NOT what happened. One slowed down, one sped up.

Exactly. That's why you don't use non-inertial frames.

Let's look at the Michelson-Morley experiment in simplistic terms for a minute. What was the "problem?" What was the suggested "solution" (explanation)?

We start with the assumption that the earth "really is" moving in a orbit about the sun.

If that's true, then we would expect light to take longer to travel between 2 co-moving points on the earth if it's travelling "with" the motion of the earth as opposed to "against" the motion of the earth. Why? Because it has to travel a different distance, depending on which way it is travelling. This was the basic assumption of Al's "simultaneity is relative" example involving the guy on the train.

But our experiment (M-M) does not produce this result. Now what?

1. One possible answer: The earth is NOT moving. It is a geocentric universe after all. For some reason, this was not deemed to be an acceptable answer. For some damn reason, contrary to the premises of SR, scientists believed that we could, and in fact had satisfactorily, determine(d) that the earth is "really moving."

2. OK, then what is the explanation? Another possibility: IF clocks slowed down by a certain degree, and if our measuring instruments shortened by a certain degree, THEN we would MEASURE the speed to be the same, even though it wasn't "really" the same.

Hence the LT.


In the MM experiement, and similar experiemnts, the clock does not change it's movement when the direction of the light changes. The clock doesn't move differently depending on how you aim the light. Therefore, time dilation is not an explanation for it's results.

The whole premise is that we are getting identical *measurements* of a phenomenon that is not "in fact" the same. This proposed solution presupposes that the underlying "reality" is different than what we empirically *measure" it to be.

Far-fetched, you say? Ad hoc, you think? Maybe.


Sounds like SR (or LR).

To me, SR is similar to the cosmological argument for the existence of God.

This argument starts by getting you to conceded that every event has a "cause" that precedes it in time.

But then it says this assumption logically leads to an unsatisfactory "infinite regression."

Therefore, it says, there must be a non-contingent "Prime Mover" or "First Cause" (which we can call God).

But notice that the solution requires that you reject the very assumption that it is based on, i.e., that every event has a pre-existing cause. But I reject that premise, then there is no force to the argument to begin with. You want me to say that "x" is true, and then use that assumption to prove that "x" is not true. See the problem?


This type of argument is called reducito ad absurdum (my apologies if you already know this). The idea is not that not-A implies A, but rather, that if A were true, it would lead to something impossible or unacceptable, so we are forced to accept not-A. I agree that there are serious problems applying RAA to real-word situations.

One Brow said...

SR does the same thing. It uses the LT. The LT presuppose that "real" motion does exist, and that "inaccurate" measurements from instrumental distortion result by a factor set forth in the LT. It supposes that there is an "objective reality" which is DIFFERENT from what is suggested by our empirical measurements.

You have confused the LT with LR. The LT are just equations that translate what one person measures into what another person measures. Equations have no preferred point of view. They are just math. They are not reality, and they do not bind reality. They are descriptive, not proscriptive, and in particular do not possess the ability to proscribe interpretations of them. The LT are fully symmetric, and recognize no preferred point of view. If you dug into the math of them, you could see this for yourself.

Based on your accepting the premise that there is "real" motion, and that there is an "objective reality" that is at variance from what we actually measure, it then tries to tell you that there is no such thing as objective reality or "real" motion. Which one is it?

Don't try to persuade me that insisting that I affirm a condition that you deny is true.


SR posits that there is relative motion, and the limits of objective reality are the descriptions of relative motion. Since the LT are symmetric, they can not insert/impose non-symmetry into SR.

As I said before, if you want to insist that the speed of light REALLY IS constant in all inertial frames of reference, THEN you would just throw the LT out the window.

When you add in the word in bold, this is one of the two principles you use to generate the LT in SR. In any inertial frame you pick, light moves at c. There is no measurement to tell you which inertial frame is really the objectively resting frame.

There would be no need for the LT to "explain" why you measure the speed of light to be the same. In that case you would "measure" it to be the same, simply because it IS the same. It could only both BE the same, and be MEASURED to be the same IF they are never any distortion to measuring instruments cause by relative motion.

Correct. In SR, the LT don't explain distortion in measuring instruments. They convert from one frame of reference to another. It may not be what Lorenz had in mind, but the LT do not care about philosophical underpinnings; they are just mathematical equations.

But of course this simplistic assertion leads to contradictions elsewhere. In effect it presupposes that nothing is ever moving, and, at the same time, that everything is moving.

Wrong. It allows conversion of the assumption one inertial frame is not moving to the assumption another inertial frame is not moving. Objects not in inertial frames definitely move, regardless. Further, the need to choose a non-moving frame first means there is always a frame that is not moving.

SR has to build in inherent conflict, where every observer denies the "reality" of the others perspective. Every observer must say only the "other" guy is moving and that only the "other guy's" clock slows down, etc.

Again, you are confusing permission with compulsion.

The logic inconsistency is obvious. Both cannot be right in their respective claims.

Neither can prove the other wrong, as long as both stay in inertial frames.

"The logic inconsistency is obvious. Both cannot be right in their respective claims."

You try to deny that there is any inconsistencies, notwithstanding the fact that virtually every scientist (along with every other rational person) denies that "each clock can actually run slower than the other."


I also deny that "each clock can actually run slower than the other." That statement makes no sense, and there is no frame in which it happens.

One Brow said...

How can you do this?

I don't. I take no responsibility for your misstatements of SR.

Only by taking a position of utter, unmitigated solipsism. There is no objective reality whatsoever. There is no "truth." The only "truth" is a subject believes is true. If two subjects believe different things, then that means there are are two absolute truths, one for each subject.

There are no absolute truths in the limited arena of which inertial frame is not moving, and you don't even believe that there is. You would have the person on earth use ECI, on Venus use VCI, etc.

This position is prima facie absurd,

Agreed.

but that's not the end of it.

It's not a part of SR at all.

SR exacerbates this inherent absurdity. The basic premise that what you "believe" to be true is, ipso facto, actually true. Now SR has to take an EXTRA step to complete it's "theory." It has to FORCE observers to "believe" things that are inherently incredible in order to create the necessary conflict. Once that forced acceptance of obvious falsity is imposed, NOW, and only now, can the theory remain consistent.

Again, you confuse permission with compulsion.

The basic premises come down to this:

1. Whatever any subject asserts to be true is, by definition, actually true.


Not true at all in SR.

2. What they "say" is true is in fact true, even if they assert that something is true, when they know it is not. It is only their assertions, no matter how insincere, that can be counted in ascertaining what is "true."

Sorry, I aint buyin.


I am glad you are not buying your strawman, that isn't SR. Perhaps I will take to calling it aintSR. How about we discuss actual SR instead?

Did you watch the "how long does it take to go 80 miles" video ...

No. The point you are trying to make with it is irrelvant.

I didn't comment on it at the time, but it appeared that you thought you were proving the "ignorance" of a Harvard physics major, by implying that he did not understand the concept of velocity.

I don't recall putting it quite in that way. What I said, to my recollection, was that he was using an OTV as the velocity of a satellite, but since a satellite is undergoing acceleration, it is not in an OTV, and using an OTV as the frame of a satellite is wrong.

One Brow said...

I have shown you, time and time again, that a "third postulate" (the clock hypothesis) holds that ONLY speed (not velocity) affects time dilation. Although the concept of an "inertial frame" is relevant to SOME aspects of SR, it is irrelevant to the question of time dilation. In that matter, acceleration is totally irrelevant.

You seem to think this is relevant to some point you have made or I have made. I'm guessing that this is because you think somehow that I believe acceleration has an effect on the amount of time dilation in SR. If so, then you have misunderstood. I am fully aware that acceleration does not affect time dilation in SR situations.

Perhaps you are not certain why I bring up acceleration so regularly then. I will state this again: the first principle of SR limits the scope of SR to measurements from inertial frames. SR makes no, zero, nada, nil, keine predictions made from the viewpoints of objects that undergo acceleration during the course of the measurement process. That means there are no, zero, nada, nil, keine SR predictions from the point of view of the navel clock in the HK experiments; no, zero, nada, nil, keine SR predictions from the viewpoint of an orbiting GPS satellite; no, zero, nada, nil, keine SR predictions from the viewpoint of the traveling astronaut. This is not because acceleration affects time dilation; it is because these viewpoints are out of the scope of SR.

You also called Louis Essen and Albert Einstein "idiots" for not agreeing with your misconception about this issue. Only YOU (well, and 99% of non-crank physicists, of course) know the real truth about this matter, eh?

We stand on the shoulders of giants. Einstein is not an idiot for struggling to figure out the on his own implications of what I was taught in a freshman physics class by a person well-versed in the topic.

LR makes accurate predictions about the time dilation of ALL objects in all frames, not just inertial ones.

Just like SR, LR requires an inertial frame from which to make its measurements. Just like SR, once you have select an inertial frame in LR, you can get the right numbers for for accelerating objects. The only difference is that SR allows you to choose any inertial frame. The invariance of the LTs makes the choice arbitrary.

It applies equally to accelerating frames. You claim that SR ONLY applies in inertial frames, and then make the absurd claim that LR is just a "special case" of SR. At best, the exact opposite is the case, i.e., SR is just a "special case" of LR. If you really understood the difference between the two, this would be obvious to you.

If you really understood SR, you would marvel at the ignorance and arrogance of what you are saying. You are claiming that a viewpoint that requires the choice of a specific inertail frame is more flexible than one that allows the choice of any inertial frame.

To refresh your memory, here is it for about the 6th or 7th time:

"Does a clock's acceleration affect its timing rate?


No.

What memory do you think you are refreshing?

As I have also repeatedly reported to you, physicists concede that the aspect of SR that is abandoned with respect to accelerating objects is the postulate pertaining to the constancy of the speed of light (as measured).

All of SR is abandoned from the viewpoint of accelerating objects. The first principle of SR requires this.

One Brow said...

This also requires the abandonment of the absurd claim that all clock retardation is "reciprocal." The "reciprocity" claim in a necessary consequence of accepting the absurd notion of "relative simultaneity."

The reciprocity is a consequence of the LT, as any web search on the topic will reveal to you.

Or, conversing, acceptance of the absurd, solipsistic notion of relative simultaneity, requires that clock retardation must be declared to be reciprocal, even though that is logically impossible as a matter of objective reality, i.e. as a matter of PHYSICS (as distinguished from math).

Describe the physics, in particular, any physical experiment you can use to disprove the reciprocity of clock retardation. So far, your attempts have been utter failures, because you insist on using accelerating reference frames.

In SR every (non-accelerating) observer in relative motion with respect to another observer MUST make conflicting assertions about the other. Unless, that is, one of them is accelerating, and then, suddenly, there is no reciprocity. Time dilation suddenly "becomes" directional.

Once they accelerate, they no longer have existed in a single inertial frame, and have no SR argument from their frame.

All this is just another way of saying that SR adopts (and in fact "turns into") LR in those cases. I know that you don't understand a word of this, so I ask myself why I bother.

A common argument from people too ignorant/arrogant to understand the first principle of SR.

Notice that this postulate is entirely independent of the LT.

In the sense that the LT are derived in part from the constancy of light.

The LT have been confirmed, this postulate has not been confirmed.

The MM experiments you mention above, among others, confirm the constancy of the speed of light. In fact, you refer to this as the reason the LT were developed to begin with. Please make up your mind here.

The postulated "reciprocity" of time dilation has NOT be been confirmed. In fact it has been effectively disproven by observations pertaining to the GPS and H-K type experiments.

Neither the GPS nor the HK experiemnts had two different clocks in different inertial frames.

Again, it is no surprise that "reciprocity" (the postulated relativity of simultaneity) has not been "confirmed," inasmuch as it is logically impossible as a matter of physics.

Describe a physical experiment that could disprove it, then.

"Just one round ago, you were discussing how SR meant two obervers in different inertial frames (such as Jack and Jill) never agreed on anything."

What they are prohibited, by fiat, from agreeing on is the question of which one is moving. With SR, Jack must say it is Jill, and Jill must say it is Jack.


Incorrect and ignorant. There is nothing in SR preventing Jack from agreeing with Jill that Jack is moving.

This further implies, for example, that they would NEVER agree on whose clock was slower. No matter what evidence to the contrary might present itself, they CANNOT agree on that. Not if they "believe" in SR, anyway. If they do, then LR is being applied, not SR. To agree implies a shared perception that some frame of reference is preferred.

LR is more restrictive than "some frame is preferred"; LR is a specific frame being preferred.

One Brow said...

That does NOT mean that there is no preferred frame for either of them. Quite the contrary. There is a preferred frame for each of them that each adamantly insists is the only correct one. The preferred frame is the one that they are in, that's all.

They each have a choice of preferred inertial frame. Jill can insist Jack is not moving, Jack can insist Jill is not moving. Either can insist both are moving.

"The preferred frame is the one that they are in, that's all."

You can't really call that "geocentric."


You also can't call it SR, unless you are ignorant of SR.

"Using LR, all "observers" will agree on the predictions: Clock A will show 101, Clock B will show 100, and clock C will show 99."

Perhaps I was not explicit enough in stating what I took to be obvious (and which I have already pointed out). When I say "using LR," I do NOT mean that you can just pick, willy-nilly, any old frame of reference, posit it to be the preferred one, and still generate predictions which comport with the observed facts. You CAN'T do that.


Incorrect and ignorant. If you pick any inertial frame of reference, at all, you get the exact same answers of 101, 100, 99. It's a property of the LT, it will happen in any theory using the LT.

Every frame of reference you choose will generate different predictions.

Incorrect and ignorant.

So the question becomes "Which (unique) frame of reference needs to be used in order to generate predictions which match the observed facts?"

Every inertial frame generates predictions that match the observed facts.

In the case of the H-K experiment, which took place in a location where the earth itself is the dominant gravitional center, the answer turns out to be the ECI.

The SCI and CMB generate the exact same predicitons as the ECI.

It is deemed to be the preferred frame. Not because of some arbitrary, whimsical free choice, but because it is the frame which gives the right answers.

The ECI is one of an infinite number of inertial frames that generate the correct answers.

You get the correct answers (predictions) when you determine how the various clocks are (absolutely) moving with respect to (relative to) the ECI, NOT with respect to how the clocks are moving relative to each other, as SR would require.

SR forbids the use of the frame of reference of acelerated clocks. SR does not require it, it forbids it.

When Newton determined the orbits of the planets in the solar system he chose, as his "preferred frame," the barycenter of the solar system. He then chose, as a "close approximation" of a stationary background, the "fixed stars." Using these components, his "celestrial mechanics" turned out to be exceedingly accurate.

Had he used the earth, or Mars, or the Sun, or any other solar object as his "preferred frame" he would have gotten completely different "answers."

Think about it.


You say that as if I would disagree. I don't. The center of the galaxy would have produced identical answers to the barycenter, but not the Sun, earth, nor Mars.